State Bank of India PO Examination 2013 Question Paper With Answer Key

State Bank of India PO Exam-2013
State Bank of India PO Examination 2013 Question Paper With Answer Key

State Bank of India PO Exam-2013 Held on 28 April, 2013

Solved Paper

Test-I

Reasoning

   Directions- (Q. 1-5) Read each statement carefully and answer the following questions-

1. Which of the following symbols should be placed in the blank spaces respectively (in the same order from left to right) in order to complete the given expression in such a manner that makes the expression ‘F > N’ and ‘U > D’ definitely false ?

F__O__U__N__D

(A)  <, <, >, =

(B)  <, =, =, >

(C)  <, =, =, <

(D)  ≥ =, =, ≥

(E)   >, >, = <

Answer: (C)

2. Which of the following symbols should be placed in the blank spaces respectively (in the same order from left to right) in order to complete the given expression in such a manner that makes the expressions ‘B > N’ as well as ‘D ≤ L’ definitely true ?

B__L__O__N__D

(A)  =, =, ≥, ≥

(B)  >, ≥, =, >

(C)  >, <, =, ≤

(D)  >, =, =, ≥

(E)   >, =, ≥, >

Answer: (D)

3. Which of the following should be placed in the blank spaces respectively (in the same order from left to right) in order to complete the given expression in such a manner that makes the expression ‘A < P’ definitely false?

__≤ __ < ___ > ___

(A)  L, N, P, A

(B)  L, A, P, N

(C)  A, L, P, N

(D)  N, A, P, L

(E)   P, N, A, L

Answer: (E)

4. Which of the following expressions will be true if the expression R > O = A > S < T is definitely true?

(A)  O > T

(B)  S < R

(C)  T > A

(D)  S = O

(E)   T < R

Answer: (B)

5. Which of the following symbols should replace the question-mark (?) in the given expression in order to make the expressions ‘P > A’ as well as ‘T ≤ L’ definitely true ?

P > L ? A ≥ N = T

(A)  ≤

(B)  >

(C)  <

(D)  ≥

(E)   Either ≤ or <

Answer: (D)

Directions –(Q. 6-10) Study the following information and answer the following questions-

   A, B, C, D, E, G and I are seen friends who study in three different standards namely 5th, 6th and 7th such that not less than two friends study in the same standard. Each Friend also has a different favourite subject namely History, Civics, English, Marathi, Hindi, Maths and Economics but not necessarily in the same order.

A likes Maths and studies in the 5th standard with only one other friend who likes Marathi. I studies  with two other friends. Both the friends who study with I like languages (Here languages include only Hindi, Marathi and English). D studies in the 6th standard with only one person and does not like Civics. E studies with only one friend. The one who likes History does not study in 5th or 6th standard. E does not like languages. C does not like English, Hindi or Civics.

6. Which of the following is I’s favourite subject?

(A)  History

(B)  Civics

(C)  Marathi

(D)  Either English or Marathi

(E)   Either English or Hindi

Answer: (A)

7. Who amongst the following studies in the 7th standard?

(A)  G

(B)  C

(C)  D

(D)  E

(E)   Either D or B

Answer: (A)

8. Which combination represents, E’s favourite subject and the standard in which he studies?

(A)  Civics and 7th

(B)  Economics and 5th

(C)  Civics and 6th

(D)  History and 7th

(E)   Economics and 7th

Answer: (C)

9. Which of the following subjects does G like?

(A)  Either Maths or Marathi

(B)  Either Hindi or English

(C)  Either Hindi or Civics

(D)  Either Hindi or Marathi

(E)   Either Civics or Economics

Answer: (B)

10. Which of the following combinations is definitely correct?

(A)  I and Hindi

(B)  G and English

(C)  C and Marathi

(D)  B and Hindi

(E)   E and Economics

Answer: (C & E)

Directions-(Q. 11-15) Study the information and answer the following questions:

   In a certain code language-

‘economics is not money’ is written as, ‘ka la ho ga’

   demand and supply economics’ is written as, ‘mo ta pa ka’

   ‘money makes only part’ is written as, ‘zi la ne ki’

   ‘demand makes supply economics’ is written as, ‘zi mo ka ta’

11. What is the code for ‘makes’ in the given code language?

(A)  mo

(B)  pa

(C)  ne

(D)  zi

(E)   ho

Answer: (D)

12. What is the code for ‘money’ in the given code language?

(A)  ga

(B)  mo

(C)  pa

(D)  ta

(E)   la

Answer: (E)

13. What may be the possible code for ‘work and money’ in the given code language?

(A)  pa ga la

(B)  pa la tu

(C)  mo la pa

(D)  tu la ga

(E)   pa la ne

Answer: (B)

14. What is the code for ‘supply’ in the given code language?

(A)  Only ta

(B)  Only mo

(C)  Either pa or mo

(D)  Only pa

(E)   Either mo or ta

Answer: (E)

15. What may be the possible code for ‘demand only more’ in the given code language?

(A)  xi ne mo

(B)  mo zi ne

(C)  ki ne mo

(D)  mo zi ki

(E)   xi ka ta

Answer: (A)

Directions-(Q. 16-20) Study the given information and answer the following question-

   When a word and number arrangement machine is given an input line of words and numbers, it arranges them following a particular rule. The following is an illustration of input and rearrangement-

(All the numbers are two digit numbers)

Input : 40 made butter 23 37 cookies salt extra 52 86 92 fell now 19

Step I : butter 19 40 made 23 37 cookies salt extra 52 86 92 fell now

Step II : cookies 23 butter 19 40 made 37 salt extra 52 86 92 fell now

Step III : extra 37 cookies 23 butter 19 40 made salt 52 86 92 fell now

Step IV : fell 40 extra 37 cookies 23 butter 19 made salt 52 86 92 now

Step V : made 52 fell 40 extra 37 cookies 23 butter 19 salt 86 92 now

Step VI : now 86 made 52 fell 40 extra 37 cookies 23 butter 19 salt 92

Step VII : salt 92 now 86 made 52 fell 40 extra 37 cookies 23 butter 19

Step VII is the last step of the above arrangement as the intended arrangement is obtained.

As per the rules followed in the given steps, find out the appropriate steps for the given input.

Input : 32 proud girl beautiful 48 55 97 rich family 61 72 17 nice life.

16. Which of the following is the third element from the left end of step VI?

(A)  beautiful

(B)  life

(C)  61

(D)  nice

(E)   17

Answer: (D)

17. How many steps will be required to complete the given input?

(A)  Five

(B)  Six

(C)  Seven

(D)  Eight

(E)   Nine

Answer: (C)

18. What is the position of ‘nice’ from the left end in the final step?

(A)  Fifth

(B)  Sixth

(C)  Seventh

(D)  Eighth

(E)   Ninth

Answer: (A)

19. Which element is third to the right of ‘family in step V?

(A)  beautiful

(B)  17

(C)  proud

(D)  97

(E)   32

Answer: (E)

20. Which of the following is step III of the given input?

(A)  Proud 72 girl 48 family 32 beautiful 17 55 97 rich 61 nice life

(B)  life 55 girl 48 family 32 beautiful 17 proud 97 rich 61 72 nice

(C)  girl 48 family 32 beautiful 17 proud 55 97 rich 61 72 nice life

(D)  family 32 beautiful 17 proud girl 48 55 97 rich 61 72 nice life

(E)   girl 48 life 55 family 32 beautiful 17 proud 97 rich 61 72 nice

Answer: (C)

Directions-(Q. 21-25) Read the information carefully and answer the following questions-

If A + B means A is the father of B

If A × B means A is the sister of B

If A $ B means A is the wife of B

If A % B means A is the mother of B

If A ÷ B means A is the son of B

21. Which among the following expressions is true if Y is the son of X is definitely false?

(A)  W % L × T × Y ÷ X

(B)  W + L × T × Y ÷ X

(C)  X + L × T × Y ÷ W

(D)  W $ X + L + Y + T

(E)   W % X + T × Y ÷ L

Answer: (D)

22. Which among the following options is true if the expression ‘I + T % J × L ÷ K’ is definitely true?

(A)  L is the daughter of T

(B)  K is the son-in-law of I

(C)  I is the grandmother of L

(D)  T is the father of J

(E)   J is the brother of L

Answer: (B)

23. Which among the given expressions indicate that M is the daughter of D?

(A)  L % R $ D + T × M

(B)  L + R $ D + M × T

(C)  L % R % D + T ÷ M

(D)  D + L $ R + M × T

(E)   L $ D ÷ R % M ÷ T

Answer: (B)

24. What should come in place of the question mark, to establish that T is the sister-in-law of Q in the expression?

R % T × ? Q + V

(A)  ÷

(B)  %

(C)  ×

(D)  $

(E)   Either $ or ×

Answer: (D)

25. What should come in place of the question mark, to establish that J is the brother of T in the expression?

J ÷ P % H ? T % L

(A)  ×

(B)  ÷

(C)  $

(D)  Either ÷ or ×

(E)   Either + or ÷

Answer: (A)

Directions-(26-30) Study the following information and answer the questions given below-

   Eight people- E, F, G, H, J, K, L and M are sitting around a circular table facing the centre. Each of them is of a different profession Chartered Accountant, Columnist, Doctor, Engineer, Financial Analyst, Lawyer, Professor and Scientist but not necessarily in the same order. F is sitting second to the left of K. The Scientist is an immediate neighbor of K. There are only three people between the Scientist and E Only one person sits between the Engineer and E. The Columnist is to the immediate right of the Engineer. M is second to the right of K. H is the Scientist. G and J are immediate neighbours of each other. Neither G nor J is an Engineer. The Financial Analyst is to the immediate left of F. The Lawyer is second to the right of the Columnist. The Professor is an immediate neighbor of the Engineer. G is second to the right of the Chartered Accountant.

26. Who is sitting second to the right of E?

(A)  The Lawyer

(B)  G

(C)  The Engineer

(D)  F

(E)   K

Answer: (B)

27. Which of the following statements is true according to the given arrangement?

(A)  The Lawyer is second to the left of the Doctor

(B)  E is an immediate neighbor of the Financial Analyst

(C)  H sits exactly between F and the Financial Analyst

(D)  Only for people sit between the Columnist and F

(E)   All of the given statements are true

Answer: (A)

28. Who amongst the following is the Professor?

(A)  F

(B)  L

(C)  M

(D)  K

(E)   J

Answer: (D)

29. Four of the following five are alike in a certain way based on the given arrangement and hence form a group. Which of the following does not belong to that group?

(A)  Chartered Accountant – H

(B)  M – Doctor

(C)  J – Engineer

(D)  Financial Analyst – L

(E)   Lawyer – K

Answer: (C)

30. What is the position of L with respect to the Scientist?

(A)  Third to the left

(B)  Second to the right

(C)  Second to the left

(D)  Third to the right

(E)   Immediate right

Answer: (B)

Directions-(Q. 31-35) In each of the questions below, two/three statements are given followed by conclusions/group of conclusions numbered I and II. You have to assume all the statements to be true even if they seem to be at variance from the commonly known facts and then decide which of the given two conclusions logically follows from the information given in the statements, Give answers-

(A) If only conclusions I follows

(B) If only conclusion II follows

(C) If either I or II follows

(D) If neither I nor II follows

(E) If both I and II follow

Statements for Q. 31 and 32-

Some square are circles.

No circle is a triangle.

No line is a square.

31. Conclusions :

(I) No triangle is a square.

(II) No line is a circle.

Answer: (D)

32. Conclusions

(I) All squares can never be triangles.

(II) Some lines are circles.

Answer: (A)

Statements for Q. 33 and 34-

All songs are poems.

All poems are rhymes.

No rhyme is a paragraph.

33. Conclusions :

(I) All rhymes are poems.

(II) All songs are rhymes.

Answer: (B)

34. Conclusions :

(I) No song is a paragraph.

(II) No poem is a paragraph.

Answer: (E)

35. Statements:

Some dews are drops.

All drops are stones.

Conclusions :

(I) Atleast some dews are stones.

(II) Atleast some stones are drops.

Answer: (E)

Directions-(Q. 36-38) Each of the questions below consists of a question of two statements numbered I and II given below it. you have to decide whether the data given in the statements are sufficient to answer the questions. Read both the statements and give answers-

(A) If the data in statement I alone are sufficient to answer the question, while the data in statement II alone are not sufficient to answer the question.

(B) If the data in statement II alone are sufficient to answer the question, while the data in statement I alone are  not sufficient to answer the question.

(C) If the data either in statement I alone or statement II alone are sufficient to answer the question.

(D) If the data neither in statement I nor II together are sufficient to answer the questions.

(E) If the data in both statement I and II together are necessary to answer the questions.

36. Six people- S, T, U, V, W and X are sitting around a circular table facing the centre. What is T’s position with respect to X?

(I) Only two people sit between U and W. X is second to the left of W. V and T are immediate neighbours of each other.

(II) T is to be immediate right of V. There are only two people between T and S. X is an immediate neighbor of S but not of V.

Answer: (E)

37. Seventeen people are standing in a straight line facing south. What is Bhavna’s position from the left end of the line?

(I) Sandeep is standing second to the left of Sheetal. Only five people stand between Sheetal and the one who is standing at the extreme right end of the line. Four people stand between Sandeep and Bhavna.

(II) Anita is standing fourth to the left of Sheetal. Less than three people are standing between Bhavna and Anita.

Answer: (E)

38. Five letters – A, E, G, N and R are arranged left to right according to certain conditions. Which letter is placed third?

(I) G is placed second to the right of A. E is to the immediate right of G. There are only two letters between R and G.

(II) N is exactly between A and G. Neither A nor G is at the extreme end of the arrangement.

Answer: (C)

Directions-(Q. 39 and 40) Read the following information carefully and answer the given question-

   The convenience of online shopping is what I like best about it. Where else can you shop even at midnight wearing your night suit? You do not have to wait in a line or wait till the shop assistant is ready to help you with your purchases. It is a much better experience as compared to going to a retail store.

-A consumer’s view

39. Which of the following can be inferred from the given information ? (An inference is something that is not directly stated but can be inferred from the given information)

(A)  One can shop online only at night

(B)  Those who are not comfortable using computers can never enjoy the experience of online shopping

(C)  All retain store provide shopping assistants to each and every customer

(D)  The consumer whose view is presented has shopped at retain stores as well as online

(E)   The consumer whose view is presented does not have any retail stores in her vicinity

Answer: (D)

40. Which of the following can be a strong argument in favour of retail store owners?

(A)  Online shopping portals offer a great deal of discounts which retail stores offer only during the sale season

(B)  One can compare a variety of products online which cannot be done at retail stores

(C)  Many online shopping portals offer the ‘cash on delivery’ feature which is for those who are skeptical about online payments

(D)  Many consumers prefer shopping at retail stores which are nearer to their houses

(E)   In online shopping the customer may be deceived as he cannot touch the product he is paying for

Answer: (E)

41. Read the following information carefully and answer the given question-

‘Pets are not allowed in the park premises’ – A notice put up at the park entrance by the authority that is responsible for maintenance of the park.

Which of the following can be an assumption according to the given information ? (An assumption is something that is supposed or taken for granted)

(A)  At least some people who visit the park have pets

(B)  This is the only park which does not allow pets

(C)  People who ignored this notice were fined

(D)  There are more than one entrances to the park

(E)   Many people have now stopped visiting the park

Answer: (A)

42. Read the following information carefully and answer the given question-

Many manufacturing companies are now shifting base to the rural areas of the country as there is a scarcity of space in urban areas. Analysts say that this shift will not have a huge impact on the prices of the products manufactured by these companies as only about 30% consumers live in urban areas.

Which of the following may be a consequence of the given information?

(A)  The prices of such products will decrease drastically in the urban areas

(B)  People living in urban areas will not be allowed to work in such manufacturing companies

(C)  These manufacturing companies had setups in the urban areas before shifting base

(D)  Those who had already migrated to the urban areas will not shift back to rural areas

(E)   The number of people migrating from rural to urban areas in search of jobs may reduce

Answer: (C)

Directions-(43-45) Read the following information carefully and answer the given questions-

   Despite repeated announcements that mobile phones were not allowed in the examination hall, three students were caught with their mobile phones.

   (a) Mobile phones nowadays have a lot of features and it is easy to cheat with their help

(b) The invigilator must immediately confiscate the mobile phones and ask the students to leave the exam hall immediately.

(c) Mobile phones are very expensive and leaving them in bags outside the exam hall of not safe.

(d) There have been incidents where students who left the exam hall early stole the mobile phones kept in the bags of the students who were writing the exam

(e) The school authorities must ask the students to leave their phones in the custody of the invigilator before the exam in order to avoid thefts of mobile phones

(f) None of the other students were carrying their phones in the exam hall

43. Which of the following among (a), (b), (e) and (f) may be the reason behind the school making such announcements before the exam?

(A)  Only (b)

(B)  Both (b) and (e)

(C)  Only (f)

(D)  Only (a)

(E)   Both (e) and (f)

Answer: (D)

44. Which of the following among (a), (b), (d) and (f) can be an immediate course of action for the invigilator?

(A)  Only (b)

(B)  Both (a) and (d)

(C)  Only (a)

(D)  Both (d) and (f)

(E)   Only (f)

Answer: (A)

45. Which of the following among (a), (b), (c) and (d) may be a strong argument in favour of the three students who were caught with the mobile phone?

(A)  Only (a)

(B)  Both (a) and (b)

(C)  Both (c) and (d)

(D)  Only (c)

(E)   Both (b) and (d)

Answer: (C)

Directions-(Q. 46-50) In each of the questions given below which one of the five answer figures on the right should come after the problem figures on the left, if the sequence were continued?

46.

Answer: (C)

47.

Answer: (A)

48.

Answer: (C)

49.

Answer: (B)

50.

Answer: (D)

Test-II

Data Analysis & Interpretation

Directions- (Q. 51-55) Study the following graph carefully to answer the questions that follow:

51. Ravinder had to purchase 45 kg of grapes from Hoshiarpur. Shopkeeper gave him discount of 4% per kg. What amount did he pay to the shopkeeper after the discount?

(A)  Rs 8,208

(B)  Rs 8,104

(C)  Rs 8,340

(D)  Rs 8,550

(E)   Rs 8,410

Answer: (A)

52. In which city is the difference between the cost of 1 kg of apple and cost of 1 kg of guava second lowest?

(A)  Jalandhar

(B)  Delhi

(C)  Chandigarh

(D)  Hoshiarpur

(E)   Ropar

Answer: (B)

53. Cost of 1 kg of guava in Jalandhar is approximately what per cent of the cost of 2 kg of grapes in Chandigarh?

(A)  66

(B)  24

(C)  28

(D)  34

(E)   58

Answer: (D)

54. What is the respective ratio between the cost of 1 kg of apples from Ropar and the cost of 1 kg of grapes from Chandigarh?

(A)  3 : 2

(B)  2 : 3

(C)  22 : 32

(D)  42 : 92

(E)   92 : 42

Answer: (C)

55. What total amount will Ram pay to the shopkeeper for purchasing 3 kg of apples and 2 kg of guavas in Delhi?

(A)  Rs 530

(B)  Rs 450

(C)  Rs 570

(D)  Rs 620

(E)   Rs 490

Answer: (C)

Directions-(Q. 56-60) Study the radar graph carefully and answer the questions that follow :

56. In which year was the difference between the number of students in university-1 and the number of students in university-2 highest?

(A)  2008

(B)  2009

(C)  2010

(D)  2011

(E)   2012

Answer: (E)

57. If 25% of the students in university-2 in the year 2010 were females, what was the number of male students in the university-2 in the same year?

(A)  11250

(B)  12350

(C)  12500

(D)  11500

(E)   11750

Answer: (A)

58. What was the per cent increase in the number of students in university-1 in the year 2011 as compared to the previous year?

(A)  135

(B)  15

(C)  115

(D)  25

(E)   35

Answer: (D)

59. What was the difference between the number of students in university-1 in the year 2010 and the number of students in university-2 in the year 2012?

(A)  Zero

(B)  5,000

(C)  15,000

(D)  10,000

(E)   1,000

Answer: (A)

60. What is the sum of the number of students in university-1 in the year 2007 and the number of students in university-2 in the year 2011 together?

(A)  50,000

(B)  55,000

(C)  45,000

(D)  57,000

(E)   40,000

Answer: (E)

Directions-(Q. 61-65) Study the graph carefully to answer the questions that follow:

61. 25% of the number of the players participating in hockey from School-5 are females. What is the number of the hockey players who are males in school-5?

(A)  15

(B)  18

(C)  30

(D)  21

(E)   27

Answer: (E)

62. Number of players participating in kho-kho from school-4 is what per cent of number of players participating in hockey from school-2?

(A)  42

(B)  48

(C)  36

(D)  40

(E)   60

Answer: (D)

63. What is the total number of players participating in hockey from all the five schools together?

(A)  324

(B)  288

(C)  342

(D)  284

(E)   248

Answer: (B)

64. What is the respective ratio between number of players participating in basketball from school-1 and the number of players participating in kho-kho from school-3?

(A)  5 : 7

(B)  7 : 9

(C)  7 : 8

(D)  9 : 7

(E)   5 : 8

Answer: (C)

65. In which school is the number of players participating in hockey and basketball together second highest?

(A)  School-1

(B)  School-2

(C)  School-3

(D)  School-4

(E)   School-5

Answer: (B)

Directions-(Q. 66-70) Study the following bar-graph carefully and answer the following questions :

66. What is the difference between Rahul’s earnings on Monday and Gita’s earnings on Tuesday?

(A)  Rs 40

(B)  Rs 20

(C)  Rs 50

(D)  Rs 30

(E)   Rs 10

Answer: (A)

67. Gita donated her earnings of Wednesday to Naveen. What was Naveen’s total earnings on Wednesday after Gita’s donation?

(A)  Rs 520

(B)  Rs 550

(C)  Rs 540

(D)  Rs 560

(E)   Rs 620

Answer: (C)

68. What is the respective ratio between Naveen’s earnings on Monday, Wednesday and Thursday?

(A)  7 : 3 : 5

(B)  8 : 6 : 5

(C)  8 : 7 : 4

(D)  9 : 5 : 4

(E)   9 : 3 : 4

Answer: (E)

69. What is the total amount earned by Rahul and Naveen together on Tuesday and Thursday together?

(A)  Rs 1,040

(B)  Rs 1,020

(C)  Rs 980

(D)  Rs 940

(E)   Rs 860

Answer: (D)

70. What is Gita’s average earnings overall the days together?

(A)  Rs 285

(B)  Rs 290

(C)  Rs 320

(D)  Rs 310

(E)   Rs 315

Answer: (B)

Directions-(Q. 71-75) Study the following pie-chart and answer the following questions :

71. 25% of employees from film production profession went on a strike. What is the number of employees from film production who have not participated in the strike?

(A)  3271

(B)  3819

(C)  3948

(D)  1273

(E)   1246

Answer: (B)

72. What is the total number of employees in engineering profession and industries together?

(A)  5698

(B)  5884

(C)  5687

(D)  5896

(E)   5487

Answer: (D)

73. In management profession three-fourth of the number of employees are female. What is the number of male employees in management profession?

(A)  1239

(B)  1143

(C)  1156

(D)  1289

(E)   1139

Answer: (E)

74. In teaching profession if three-fifth of the teachers are not permanent, what is the number of permanent teachers in the teaching profession?

(A)  1608

(B)  1640

(C)  1764

(D)  1704

(E)   1686

Answer: (A)

75. What is the difference between the total number of employees in teaching and medical profession together and the number of employees in management profession?

(A)  6770

(B)  7700

(C)  6700

(D)  7770

(E)   7670

Answer: (C)

Directions-(Q. 76-80) Study the table carefully to answer the questions that follow :

76. What is the average electricity bill paid by Manu over all the five months together?

(A)  Rs 183

(B)  Rs 149

(C)  Rs 159

(D)  Rs 178

(E)   Rs 164

Answer: (C)

77. What is the total amount of bill paid by Dev in the month of June for all the four commodities?

(A)  Rs 608

(B)  Rs 763

(C)  Rs 731

(D)  Rs 683

(E)   Rs 674

Answer: (C)

78. In which months respectively did Manu pay the second highest mobile phone bill and the lowest electricity bill?

(A)  April and June

(B)  April and May

(C)  March and June

(D)  March and May

(E)   July and May

Answer: (D)

79. What is the respective ratio between the electricity bill paid by Manu in the month of April and the mobile phone bill paid by Ravi in the month of June?

(A)  27 : 49

(B)  27 : 65

(C)  34 : 49

(D)  135 : 184

(E)   13 : 24

Answer: (A)

80. What is the difference between the mobile phone bill paid by Ravi in the month of May and the laundry bill paid by Dev in the month of March?

(A)  Rs 180

(B)  Rs 176

(C)  Rs 190

(D)  Rs 167

(E)   Rs 196

Answer: (A)

Directions-(Q. 81-85) Study the following table carefully and answer the questions that follows :

81. Distance between which two stations is second lowest?

(A)  Nadiad Jn. to Ahmedabad

(B)  Anand Jn. to Nadiad Jn.

(C)  Dadar to Vasai Road

(D)  Anand Jn. to Vadodara

(E)   Vasai Road to Surat

Answer: (B)

82. What is the distance travelled by the train from Surat to Nadiad Jn.?

(A)  176 km

(B)  188 km

(C)  183 km

(D)  193 km

(E)   159 km

Answer: (C)

83. How much time does the train take to reach Ahmedabad after departing from Anand Jn. (including the halt time)?

(A)  1 hr. 59 min.

(B)  1 hr. 17 min.

(C)  1 hr. 47 min.

(D)  1 hr. 45 min.

(E)   1 hr. 15 min.

Answer: (E)

84. What is the respective ratio between the number of passengers boarding from Vasai Road and from Ahemdabad in the train?

(A)  21 : 17

(B)  13 : 9

(C)  21 : 19

(D)  15 : 13

(E)   13 : 15

Answer: (A)

85. If halt time (stopping time) of the train at Vadodara is decreased by 2 minutes and increased by 23 minutes at Ahemdabad. At what time will be the train reach Bhuj?

(A)  6.10 a.m.

(B)  6.01 p.m.

(C)  6.05 a.m.

(D)  6.50 p.m.

(E)   6.07 p.m.

Answer: (B)

Directions-(Q. 86-90) Study the table carefully to answer the questions that follow :

86. What is the average maximum temperature of Beijing over all the months together?

(A)  8.4°C

(B)  9.6°C

(C)  7.6°C

(D)  9.2°C

(E)   8.6°C

Answer: (E)

87. What is the difference between the maximum temperature of Ontario on 1st November and the minimum temperature of Bhuj on 1st January?

(A)  3°C

(B)  18°C

(C)  15°C

(D)  9°C

(E)   11°C

Answer: (E)

88. In which month respectively the maximum temperature of Kabul is second highest and minimum temperature of Sydney is highest?

(A)  1st October and 1st January

(B)  1st October and 1st November

(C)  1st December and 1st January

(D)  1st September and 1st January

(E)   1st December and 1st September

Answer: (A)

89. What is the respective ratio between the minimum temperature of Beijing on 1st September and the maximum temperature on Ontario on 1st October?

(A)  3 : 4

(B)  3 : 5

(C)  4 : 5

(D)  1 : 5

(E)   1 : 4

Answer: (B)

90. In which month (on 1st day) is the difference between maximum temperature and minimum temperature of Bhuj second highest?

(A)  1st September

(B)  1st October

(C)  1st November

(D)  1st December

(E)   1st January

Answer: (C)

Directions-(Q. 91-95) Study pie-chart and table carefully to answer the questions that follow :

91. What is the difference between the number of diesel engine cars in state-2 and the number of petrol engine cars in state-4?

(A)  159

(B)  21

(C)  28

(D)  34

(E)   16

Answer: (B)

92. What is the difference between the total number of cars in state-3 and the number of petrol engine cars in state-2?

(A)  96

(B)  106

(C)  112

(D)  102

(E)   98

Answer: (E)

93. What is the average number of petrol engine cars in all the states together?

(A)  86.75

(B)  89.25

(C)  89.75

(D)  86.25

(E)   88.75

Answer: (B)

94. Number of petrol engine cars in state-3 is what per cent more than the number of diesel engine cars in state-1?

(A)  100

(B)  200

(C)  300

(D)  125

(E)   225

Answer: (A)

95. If 25% of diesel engine cars in state-3 are AC and remaining cars are non-AC, what is the number of diesel engine cars in state-3 which are non-AC?

(A)  75

(B)  45

(C)  95

(D)  105

(E)   35

Answer: (D)

96. A die is thrown twice. What is the probability of getting a sum 7 from both the throws?

(A)  5/18

(B)  1/18

(C)  1/9

(D)  1/6

(E)   5/36

Answer: (D)

97. A bag contains 7 blue balls and 5 yellow balls. If two balls are selected at random, what is the probability that none is yellow ?

(A)  5/33

(B)  5/22

(C)  7/22

(D)  7/33

(E)   7/66

Answer: (E)

Directions-(Q. 98-100) Study the information carefully to answer the questions :

   In a team there are 240 members (males and females). Two-third of them are males. Fifteen per cent of males are graduates. Remaining males are non-graduates. Three-fourth of the females are graduates. Remaining females are non-graduates.

98. What is the sum of the number of females who are graduates and the number of males who are non-graduates?

(A)  184

(B)  96

(C)  156

(D)  84

(E)   196

Answer: (E)

99. What is the ratio between the total number of males and the number of females who are non-graduates?

(A)  6 : 1

(B)  8 : 1

(C)  8 : 3

(D)  5 : 2

(E)   7 : 2

Answer: (B)

100. What is the difference between the number of females who are non-graduates and the number of males who are graduates?

(A)  2

(B)  24

(C)  4

(D)  116

(E)   36

Answer: (C)

Test-III

General Awareness, Marketing & Computer

101. Every year March 20 is celebrated as what day?

(A)  World Sparrow Day

(B)  International Women’s Day

(C)  World Cuckoo Day

(D)  International Child Day

(E)   International Mother’s Day

Answer: (A)

102. Invisible Export means export of-

(A)  Services

(B)  Prohibited goods

(C)  Restricted goods

(D)  Goods are per OGL list

(E)   Other than those given as options

Answer: (A)

103. One of the famous Indian Sportsperson released his/her autobiography- ‘Playing to Win’ in November 2012. Name the sportsperson correctly-

(A)  Saina Nehwal

(B)  Mary Kom

(C)  Yuvraj Singh

(D)  Sachin Tendulkar

(E)   Sushil Kumar

Answer: (A)

104. The NRIs while investing in the equity of a company cannot buy more than prescribed percentage of the paid up capital of that company. What is the prescribed percentage?

(A)  2%

(B)  3%

(C)  4%

(D)  5%

(E)   6%

Answer: (D)

105. Government as part of the recapitalization plan, infused capital in State Bank of India recently. Indicate the approximate capital infusion-

(A)  Rs 500 cr.

(B)  Rs 1000 cr.

(C)  Rs 1,500 cr.

(D)  Rs 2,000 cr.

(E)   Rs 3,000 cr.

Answer: (E)

106. Muhammad Yunus, the recipient of the Nobel Peace Prize 2006 is the exponent of which of the following concepts in the field of banking?

(A)  Core Banking

(B)  Micro Credit

(C)  Retail Banking

(D)  Real Time Gross Settlement

(E)   Internet Banking

Answer: (B)

107. Banks in India are required to maintain a portion of their demand and time liabilities with the Reserve Bank of India. This portion is called-

(A)  Statutory Liquidity Ratio

(B)  Cash Reserve Ratio

(C)  Bank Deposit

(D)  Reverse Repo

(E)   Government Securities

Answer: (B)

108. The European Union has adopted which of the following as a common currency?

(A)  Dollar

(B)  Dinar

(C)  Yen

(D)  Euro

(E)   Peso

Answer: (D)

109. Who is the Captain of Australian Cricket Team, which currently (March 2013) visited India?

(A)  Michael Clarke

(B)  Shane Watson

(C)  Shane Warne

(D)  Michael Hussey

(E)   Ricky Ponting

Answer: (A)

110. Who is the author of the book ‘Women of Vision’?

(A)  Ravinder Singh

(B)  Preeti Shenoy

(C)  Amish Tripathi

(D)  Durjoy Dutta

(E)   Alam Srinivas

Answer: (E)

111. Which of the following term is associated with the game of Cricket?

(A)  Bouncer

(B)  Love

(C)  Goal

(D)  Mid Court

(E)   Collision

Answer: (A)

112. The Holidays for the Banks are declared as per-

(A)  Reserve Bank Act

(B)  Banking Regulation Act

(C)  Negotiable Instruments Act

(D)  Securities and Exchange Board of India Act

(E)   Companies Act

Answer: (C)

113. In banking business, when the borrowers avail a Term Loan, initially they are given a repayment holiday and this is referred as-

(A)  Subsidy

(B)  Interest Water

(C)  Re-phasing

(D)  Interest concession

(E)   Moratorium

Answer: (E)

114. One of IT company from India has become the first Indian Company to trade on NYSE Euronext London and Paris markets from February 2013 onwards. Which is this company?

(A)  Wipro Infotech Ltd.

(B)  L & T Infotech

(C)  HCL Technologies LTD.

(D)  Infosys Technologies Limited

(E)   Polaris Financial Technology Ltd.

Answer: (D)

115. Banking Ombudsman is appointed by-

(A)  Government of India

(B)  State Governments

(C)  RBI

(D)  ECGC

(E)   Exim Bank

Answer: (C)

116. BSE (Bombay Stock Exchange), the oldest stock exchange in Asia has joined hands with one more International index in February 2013. This association has resulted in change of name of BSE index. What is the change of name effected?

(A)  Dow Jones BSE Index

(B)  NASDAQ BSE Index

(C)  S & P BSE Index

(D)  Euronext BSE Index

(E)   Other th an those given as options

Answer: (C)

117. Interest on Savings deposit now-a-days is-

(A)  Fixed by RBI

(B)  Fixed by the respective Banks

(C)  Fixed by the Depositors

(D)  Fixed as per the contract between Bank and the Consumer Court

(E)   Not pay by the Bank

Answer: (B)

118. The customers’ by opening and investing in the Tax Saver Deposit Account Scheme in a bank would get benefit under-

(A)  Sales Tax

(B)  Customs Duty

(C)  Excise Duty

(D)  Professional Tax

(E)   Income Tax

Answer: (E)

119. Pre-shipment finance is provided by the banks only to-

(A)  Credit Card Holders

(B)  Students aspiring for further studies

(C)  Brokers in equity market

(D)  Village Artisans

(E)   Exporters

Answer: (E)

120. A non-performing asset is-

(A)  Money at call and short notice

(B)  An asset that ceases to generate income

(C)  Cash balance in till

(D)  Cash balance with RBI

(E)   Balance with other banks

Answer: (B)

121. Interest below which a bank is not expected to lend to customers is known as-

(A)  Deposit Rate

(B)  Base Rate

(C)  Prime Lending Rate

(D)  Bank Rate

(E)   Discount Rate

Answer: (B)

122. Government usually classifies its expenditure in term of planned and non-planned expenditure. Identify, which is the correct definition of planned expenditure?

(A)  It represents the expenditure of all the State Governments

(B)  It represents the total expenditure of the Central Government

(C)  It is the expenditure which is spent through centrally sponsored programmes and flagship schemes of Government

(D)  It represents the expenditure incurred on Defence

(E)   Other than those given as options

Answer: (E)

123. Which of the following organization is made specifically responsible for empowering Micro, Small and Medium enterprises in India?

(A)  NABARD

(B)  RBI

(C)  SIDBI

(D)  ECGC

(E)   SEBI

Answer: (C)

124. The Union Budget for 2013-14 proposed by the Finance Minister on February 28, 2013 announced introduction of new variety of bonds by the Government. What is the name of these bonds?

(A)  Deep discounts bonds

(B)  Zero Coupon bonds

(C)  Bullet Bonds

(D)  Inflation Indexed Bonds

(E)   Inflation Variable Bonds

Answer: (D)

125. RBI has released its guidelines for entry of new banks in the private sector in the month of February 2013. One of the norms is at least a prescribed percentage of branches of new bank should be set in unbanked rural centres with a population of upto 9,999 people. What is the percentage of such branches prescribed in the norms?

(A)  10%

(B)  15%

(C)  18%

(D)  25%

(E)   Other than those given as options

Answer: (D)

126. A joystick is primarily used to/for-

(A)  Control sound on the screen

(B)  Computer gaming

(C)  Enter text

(D)  Draw pictures

(E)   Print text

Answer: (B)

127. The CPU comprises of control, Memory and ……… units.

(A)  Microprocessor

(B)  Arithmetic/Logic

(C)  Output

(D)  ROM

(E)   Input

Answer: (B)

128. Which of the following uses a handheld operating system?

(A)  A super computer

(B)  A personal computer

(C)  A laptop

(D)  A mainframe

(E)   A PDA

Answer: (E)

129. To display the contents of a folder in Windows Explorer you should-

(A)  click on it

(B)  collapse it

(C)  name it

(D)  give it password

(E)   rename it

Answer: (A)

130. ‘C’ in CPU denotes-

(A)  Central

(B)  Common

(C)  Convenient

(D)  Computer

(E)   Circuitry

Answer: (A)

131. …………… is the most important/powerful computer in a typical network.

(A)  Desktop

(B)  Network Client

(C)  Network server

(D)  Network station

(E)   Network switch

Answer: (C)

132. Which is not a storage device?

(A)  A CD

(B)  A DVD

(C)  A Floppy disk

(D)  A Printer

(E)   A Hard disk

Answer: (D)

133. Which of the following refers to the rectangular area for displaying information and running programs?

(A)  Desktop

(B)  Dialog box

(C)  Menu

(D)  Window

(E)   Icon

Answer: (B)

134. ………….. devices convert human-under-standable data and programs into a form that the computer can process.

(A)  Printing

(B)  Output

(C)  Solid state

(D)  Monitor

(E)   Input

Answer: (E)

135. The software that is used to create text-based documents are referred to as-

(A)  DBMS

(B)  suites

(C)  spreadsheets

(D)  presentation software

(E)   word processors

Answer: (E)

136. …………….. is a windows utility program that locates and eliminates unnecessary fragments and rearranges files and unused-disk space to optimize operations.

(A)  Backup

(B)  Disk Cleanup

(C)  Disk Defragmenter

(D)  Restore

(E)   Disk Restorer

Answer: (C)

137. Which of the following refers to too much electricity and may cause a voltage surge?

(A)  Anomaly

(B)  Shock

(C)  Spike

(D)  Virus

(E)   Splash

Answer: (C)

138. A(n) ………. appearing on a web page opens another document when clicked.

(A)  anchor

(B)  URL

(C)  hyperlink

(D)  reference

(E)   heading

Answer: (C)

139. A successful marketing person requires one of the following qualities-

(A)  Empathy

(B)  Sympathy

(C)  Insistence

(D)  Aggressiveness

(E)   Pride

Answer: (B)

140. Market segmentation can be resorted to by dividing the target group as per-

(A)  Income levels of customers

(B)  Age of the employees

(C)  Needs of the sales persons

(D)  Marketing skills of the employees

(E)   Size of the Organization

Answer: (A)

141. Innovation in marketing is same as-

(A)  Abbreviation

(B)  Communication

(C)  Creativity

(D)  Aspiration

(E)   Research work

Answer: (E)

142. Effective Selling Skills depends on-

(A)  Number of languages known to the DSA

(B)  Data on marketing staff

(C)  Information regarding IT market

(D)  Knowledge of related markets

(E)   Ability to talk fast

Answer: (D)

143. A Direct Selling Agent (DSA) is required to be adept in-

(A)  Surrogate marketing

(B)  Training skills

(C)  Communication skills

(D)  Market Research

(E)   OTC Marketing

Answer: (C)

144. Leads can be best sourced from-

(A)  Foreign customers

(B)  Yellow pages

(C)  Dictionary

(D)  List of vendors

(E)   Local supply chains

Answer: (B)

145. Value added services means-

(A)  Low cost products

(B)  High cost products

(C)  At par services

(D)  Additional services for the same cost

(E)   Giving discounts

Answer: (D)

146. Post-sales activities includes-

(A)  Sales presentation

(B)  Customer Feedback

(C)  Customer identification

(D)  Customer apathy

(E)   Product design

Answer: (B)

147. The target market for Debit Cards is-

(A)  All existing account-holders

(B)  All agriculturists

(C)  All DSAs

(D)  All vendors

(E)   All outsourced agents

Answer: (A)

148. The competitive position of a company can be improved by-

(A)  increasing the selling price

(B)  reducing the margin

(C)  ignoring competitors

(D)  increasing the cost price

(E)   understanding and fulfilling customers’ needs

Answer: (E)

149. A good Brand can be built up by way of-

(A)  Customer grievances

(B)  Break-down of IT support

(C)  Old age

(D)  Large number of products

(E)   Consistent offering of good services

Answer: (E)

150. The ‘USP’ of a product denotes-

(A)  Usefulness of the product

(B)  Drawbacks of a product

(C)  Main functions

(D)  Number of allied products available

(E)   High selling features of a product

Answer: (E)

Test-IV

English Language

   Directions-(Q. 151-160) Read the following passage carefully and answer the questions given below it. Certain words have been printed in bold to help you located them while answering some of the questions.

   Financial Inclusion (FI) is an emerging priority for banks that have nowhere else to go to achieve business growth. The viability of FI business is under question, because while banks and their delivery partners continue to make investments, they haven’t seen commensurate returns. In markets like India, most programs are focused on customer on-boarding, an expensive process which people often find difficult to afford, involving issuance of smart cards to the customers. However, large scale customer acquisition hasn’t translated into large scale business, with many accounts lying dormant and therefore yielding no return on the bank’s investment. For the same reason, Business Correspondent Agents who constitute the primary channel for financial inclusion are  unable to pursue their activity as a full-time job. One major reason for this state of events is that the customer on-boarding process is often delayed after the submission of documents (required to validate the details of the concerned applicant) by the applicant and might take as long as two weeks. By this time the initial enthusiasm of applicants fades away. Moreover, the delivery partners don’t have the knowledge and skill to propose anything other than the most basic financial products to the customer and hence do not serve their banks’ goal of expanding the offering in unbanked markets.

Contrary to popular perception, the inclusion segment is not a singular impoverished, undifferentiated mass and  it is important to navigate its diversity to identify the right target customers for various programs. Rural markets do have their share of rich people who do not use banking services simply because they are inconvenient to access or have low perceived value. At the same time, urban markets, despite a high branch density, have multitude of low wage earners outside the financial net. Moreover, the branch timings of banks rarely coincide with the off-work hours of the labour class.

Creating affordability is crucial in tapping the unbanked market. No doubt pricing is a tool, but banks also need to be innovative in rightsizing their proposition to convince customers that they can derive big value even from small amounts. One way of doing this is to show the target audience that a bank account is actually a lifestyle enabler, a convenient and safe means to send money to family or make a variety of purchases. Once banks succeed in hooking customer with this value proposition they must sustain their interest by introducing a simple and intuitive user application, ubiquitous access over mobile and other touch points, and adopting a banking mechanism which is not only secure but also reassuring to the customer. Technology is the most important element of financial inclusion strategy and an enable of all others. The choice of technology is therefore a crucial decision, which could make or mar the agenda. Of the various selection criteria, cost is perhaps the most important. This certainly does not mean buying the cheapest package, but rather choosing that solution which by scaling transactions to huge volumes reduces per unit operating cost. An optimal mix of these strategies would no doubt offer an innovative means of expansion in the unbanked market.

151. According to the passage, for which of the following reasons do the delivery partners fail to serve their bank’s goal to expand in the unbanked markets?

(1) They do not have adequate client base to sell their financial products.

(2) They do not have adequate knowledge and skills to explain anything beyond basic financial products to the customers.

(3) They do not have the skills to operate advanced technological aids that are a prerequisite to tap the unbanked market.

(A)  Only 2

(B)  Only 3

(C)  All 1, 2 and 3

(D)  Only 1

(E)   Both 2 and 3

Answer: (A)

152. Which of the following facts is true as per the passage?

(A)  People from rural areas have high perceived value of banking services

(B)  Cost is not a valid criterion for technological package selection for financial inclusion initiatives

(C)  The inclusion segment is a singular impoverished, undifferentiated mass

(D)  The branch timings of banks generally do not coincide with the off-work hours of the labour lass in urban markets

(E)   All the given statements are true

Answer: (D)

153. In the passage, the author has specified which of the following characteristics of the customer on boarding process?

(A)  It involves collection of documents from the applicants in order to validate their details

(B)  It involves issuance of smart cards to the customers

(C)  It suffers from latency as it takes a long time after submission of documents by the customer

(D)  It is an expensive process which people find difficult to afford

(E)   All of the given characteristics have been specified

Answer: (E)

154. According to the passage, which of the following ways may help banks to sustain the interest of their customers after hooking them?

(1) Adoption of baking mechanism which is not secure but reassuring to the customers.

(2) Increasing the number of delivery partners in rural market.

(3) Introduction of a simple and intuitive user application.

(A)  Only 1

(B)  Only 3

(C)  Only 2

(D)  All 1, 2 and 3

(E)   Both 1 and 3

Answer: (E)

155. According to the passage, for which of the following reasons is the viability of financial inclusion under question?

(A)  The banks always prefer the cheapest package (to cut cost) while making a choice of technology to be used

(B)  The Business Correspondent Agents are highly demotivated to pursue their activity as a full-time job

(C)  The investments made by banks and its delivery partners are not yielding equal amount of returns

(D)  The banks do not have adequate number of delivery partners required to tap the unbanked market

(E)   The banks do not have adequate manpower to explore the diversity of the unbanked marked and thereby identify the right target customers for various programs

Answer: (C)

156. What did the author try to highlight in the passage?

(1) The ailing condition of financial inclusion business at present.

(2) Strategies that may help bank to expand in the unbanked market.

(3) Role of government in modifying the existing financial inclusion policies.

(A)  Both 1 and 2

(B)  All, 1, 2 and 3

(C)  Only 3

(D)  Only 1

(E)   Only 2

Answer: (A)

Directions-(Q. 157 and 158) Choose the word which is most similar in meaning to the word printed in bold as used in the passage.

157. Ubiquitous

(A)  Quintessential

(B)  Popular

(C)  Omnipresent

(D)  Simplified

(E)   Abnormal

Answer: (C)

158. Multitude

(A)  Impoverished

(B)  Handful

(C)  Acknowledged

(D)  Plenty

(E)   Solitude

Answer: (D)

Directions-(Q. 159 and 160) Choose the word which is most opposite in meaning to the word printed in bold as used in the passage.

159. Delayed

(A)  Perturbed

(B)  Popularized

(C)  Expedited

(D)  Stabilised

(E)   Repressed

Answer: (C)

160. Dormant

(A)  Emaciated

(B)  Pertinent

(C)  Cornered

(D)  Rejected

(E)   Active

Answer: (E)

Directions-(Q. 161-170) Read the following passage carefully and answer the questions given below it. Certain words have been printed in bold to help you locate them while answering some of the questions.

   The evolution of Bring Your Own Device (BYOD) trend has been as profound as it has been rapid. It represents the more visible sign that the boundaries between personal life and work life are blurring. The 9 am – 5 pm model of working solely from office has become archaic and increasingly people are working extended hours from a range of locations. At the very heart of this evolution is the ability to access enterprise networks from anywhere and anytime. The concept of cloud computing serves effectively to extend the office out of office. The much heralded benefit of BYOD is greater productivity. However, recent research has suggested that this is the greatest myth of BYOD and the reality is that BYOD in practice poses new challenges that may outweigh the benefits. A worldwide survey commissioned by Fortinet chose to look at attitudes towards BYOD and security from the users’ point of view instead of the IT managers. Specifically the survey was conducted in 15 territories on a group of graduate employees in their early twenties because they represent the first generation to enter the work-place with an expectation of own device use. Moreover, they also represent tomorrow’s influencers and decision makers. The survey findings reveal that for financial organizations, the decision to embrace BYOD is extremely dangerous. Larger organizations will have mature IT strategies and policies in place. But what about smaller financial businesses? They might not have such well developed strategies to protect confidential data.

Crucially, within younger employee group, 55% of the people share an expectation that they should be allowed to use their own devices in the workplace or for work purposes. With this expectation comes the very real risk that employees may consider contravening company policy banning the use of own devices. The threats posed by this level of subversion cannot be overstated. The survey casts doubt on the idea of BYOD leading to greater productivity by revealing the real reason people want to use their own devices. Only 26% of people in this age group cite efficiency as the reason they want to use their own devices, while 63% admit that the main reason is so they have access to their favourite applications. But with personal applications so close to hand, the risk to the business must surely include distraction and time wasting. To support this assumption 46% of people polled acknowledged time wasting as the greatest threat to the organization, while 42% citing greater exposure to theft or loss of confidential data. Clearly, from a user perspective there is great deal of contradiction surrounding BYOD and there exists an under-current of selfishness where users expect to use their own devices, but mostly for personal interest. The recognized the risks to the organization but are adamant that those risks are worth taking.

161. Which of the following is not true about BYOD?

(A)  BYOD enables employees to access enterprise network from anywhere and anytime

(B)  Due to evolution of BYOD trend the 9 am – 5 pm model of working solely from office has become outdated

(C)  Recent research has confirmed that BYOD boosts organizational productivity

(D)  The concept of cloud computing facilitates the BYOD trend

(E)   All the given facts are true

Answer: (E)

162. According to the passage, the expectation of younger employees that they should be allowed to use their own devices in the workplace, entails which of the following risks?

(1) Younger employees may deliberately transfer confidential data of their companies to rivals if they are not allowed to use their own devices for work purpose.

(2) Younger employees may strongly feel like leaving the company if it prevents usage of own device and join some other company that does not have such stringent policies.

(3) Younger employees may consider flouting company policy prohibiting usage of their own devices in the workplace or for work purposes.

(A)  Only 3

(B)  Only 2

(C)  Both 1 and 3

(D)  Only 1

(E)   All, 1, 2 and 3

Answer: (B)

163. According the passage, for which of the following reasons did Fortinet conduct the survey on a group of graduate employees in their early twenties?

(1) As this group represents the future decision makers.

(2) As this group represents the first generation who entered the workforce with a better understanding of sophisticated gadgets.

(3) As this group represents the first generation on enter the workplace expecting that they can use their own devices for work purpose.

(A)  All 1, 2 and 3

(B)  Only 3

(C)  Both 1 and 3

(D)  Only 1

(E)   Only 2

Answer: (C)

164. According to the passage, why would the decision to embrace BYOD prove dangerous to smaller financial businesses?

(A)  Their employees have poor knowledge about their devices which in turn pose a threat to the confidential data of the organization

(B)  Their employees are more vulnerable to misplacement of devices

(C)  They may lack mature IT strategies and policies required to protect confidential data

(D)  They cannot afford to deal with damage liability issues of employee-owned devices

(E)   Their employees have a tendency to change jobs frequently

Answer: (A)

165. What is/are the author’s main objective(s) in writing the passage?

(1) To break the myth that BYOD promotes employees efficiency and organizational productivity.

(2) To suggest ways to control subversion across levels to corporate chain of command.

(3) To throw light upon the fact that employees even after knowing the risks involved, prefer to use their own devices for work purpose mostly for personal benefits.

(A)  Both 1 and 3

(B)  All 1, 2 and 3

(C)  Only 3

(D)  Only 1

(E)   Only 2

Answer: (A)

166. According to the findings of the survey conducted by Fortinet, why do majority of employees prefer using their own devices for work purpose?

(A)  As they often find that the devices provided by the company lack quality

(B)  As they have access to their favourite applications while working

(C)  As majority of them believe that output doubles when they use their own devices for work purpose

(D)  As handling data from their own devices reinforces their sense of responsibility

(E)   As it helps them create a brand of their own

Answer: (B)

Directions-(Q. 167 and 168) Choose the word which is most similar in meaning to the word printed in bold as used in the passage.

167. Outweigh

(A)  Control

(B)  Venture

(C)  Perish

(D)  Determine

(E)   Surpass

Answer: (E)

168. Heralded

(A)  Suspected

(B)  Publicised

(C)  Dragged

(D)  Objective

(E)   Benefit

Answer: (A)

Directions-(Q. 169 and 170) Choose the word which is most opposite in meaning to the word printed in bold as used in the passage.

169. Subversion

(A)  Compliance

(B)  Sanity

(C)  Popularity

(D)  Destabilisation

(E)   Clarity

Answer: (A)

170. Embrace

(A)  Contradict

(B)  Disobey

(C)  Curtail

(D)  Reject

(E)   Obscure

Answer: (D)

Directions-(Q. 171-175) Read each sentence to find out whether there is any grammatical mistake/error in it. The error if any, will be one part of the sentence. Mark the number of that part with error as  your answer. If there is ‘No error’, mark (E).

171. Equal opportunities for advancement (A) / across the length and breadth (B)/ of an organization will (C)/ keep many problems away. (D)/ No error (E)

Answer: (E)

172. A customized data science degree (A) / is yet to become (B)/ a standard programme (C)/ to India’s premier educational institutes. (D)/ No error (E)

Answer: (D)

173. There cannot be any situation where (A) / somebody makes money in an asset (B)/ located in India and does not pay tax (C)/ either to India or to the country of this origin. (D)/ No error (E)

Answer: (D)

174. India has entered a downward spiral (A) / where the organized, productive (B)/ and law abide sectors are subject to (C)/ savage amounts of multiple taxes. (D)/ No error (E)

Answer: (C)

175. The bank may have followed (A) / an aggressive monetary tightening policy (B)/ but its stated aim of (C)/ curbing inflation have not been archieved. (D)/ No error (E)

Answer: (D)

Directions-(Q. 176-180) Each question below has two blanks, each blank indicating that something has been omitted. Choose the set of words for each blank that best fits the meaning of the sentence as a whole.

176. Brands ……. decision-simplicity strategies make full use of available information to ………. where consumers are non the path of decision making and direct them to the best market offers.

(A)  diluting, divulge

(B)  tempting, maintain

(C)  imputing, overdrive

(D)  pursuing, assess

(E)   employing, trust

Answer: (A)

177. If India is ……….. on protecting its resources international business appears equally ………. to safeguard its profit.

(A)  dreaded, fragile

(B)  stubborn, weak

(C)  bent, determined

(D)  approaching, settled

(E)   obsessed, prepared

Answer: (C)

178. The numbers ………. by the legitimate online music service providers indicate that a growing number of users are …….. to buy music.

(A)  morphed, ignoring

(B)  labelled, thriving

(C)  figured, fanatic

(D)  painted, interested

(E)   touted, willing

Answer: (C)

179. Lack of financing options, ……… with HR and technological ……….., make small and medium enterprises sector the most vulnerable component of our economy.

(A)  except, loophole

(B)  coupled, challenges

(C)  armed, benefits

(D)  registered, strategies

(E)   strengthened, facilities

Answer: (B)

180. When you want to digitalize a city ………. with millions, you don’t bet ……. the odds.

(A)  proceeding, into

(B)  teeming, against

(C)  undergoing, adhere

(D)  dangling, for

(E)   falling, above

Answer: (B)

Directions-(Q. 181-185) Rearrange the following six sentences 1, 2, 3, 4, 5 and 6 in the proper sequence to form a meaningful paragraph; then answer the questions given below them.

(1) The group desired to enhance the learning experience in schools with an interactive digital medium that could be used within and outside the class-room.

(2) Then the teacher can act on the downloaded data rather than collect it from each and every student and thereby save his time and effort.

(3) Edutor, decided the group of engineers, all alumni of the Indian Institute of technology, when they founded Edutor Technologies in August 2009.

(4) They can even take tests and submit them digitally using the same tablets and the teachers in turn can download the tests using the company’s cloud services.

(5) With his desire they created a solution that digitizers school textbooks and other learning material so that students no longer need to carry as many books to school and back as before, but can access their study material on their touch-screen tablets.

(6) A mechanic works on motors and an accountant has his computer. Likewise, if a student has to work on a machine or device, what should it be called?

181. Which of the following sentences should be the SIXTH (LAST) after rearrangement?

(A)  1

(B)  6

(C)  5

(D)  2

(E)   4

Answer: (D)

182. Which of the following sentences should be the FOURTH after rearrangement?

(A)  1

(B)  6

(C)  5

(D)  2

(E)   3

Answer: (C)

183. Which of the following sentences should be the FIFTH after rearrangement?

(A)  1

(B)  4

(C)  3

(D)  5

(E)   6

Answer: (B)

184. Which of the following sentences should be the FIRST after rearrangement?

(A)  6

(B)  4

(C)  1

(D)  3

(E)   5

Answer: (A)

185. Which of the following sentences should be the THIRD after rearrangement?

(A)  1

(B)  2

(C)  4

(D)  5

(E)   6

Answer: (A)

Directions-(Q. 186-190) In each of the following questions four words are given of which two words are most nearly the same or opposite in meaning. Find the two words which are most nearly the same or opposite in meaning and indicate the number of the correct letter combination, by darkening the appropriate oval in your answer sheet.

186. (1) Resilient       (2) Subsidiary

(3) Rigid             (4) Arrogant

(A)  2-4

(B)  1-2

(C)  3-2

(D)  1-3

(E)   1-4

Answer: (D)

187. (1) Verifiable (2) Intentional

(3) Compliant     (4) Inadvertent

(A)  3-2

(B)  2-4

(C)  1-2

(D)  1-3

(E)   3-4

Answer: (B)

188. (1) Arcane (2) Dreadful

(3) Concocted    (4) Mysterious

(A)  1-4

(B)  1-2

(C)  4-3

(D)  3-2

(E)   2-4

Answer: (A)

189. (1) Stubborn (2) Placid

(3) Impede         (4) Obstinate

(A)  4-1

(B)  2-3

(C)  3-4

(D)  2-1

(E)   1-3

Answer: (A)

190. (1) Succinct (2) Curt

(3) Correct          (4) Stoic

(A)  1-3

(B)  2-4

(C)  4-1

(D)  3-2

(E)   1-2

Answer: (E)

Directions-(Q. 191-200) In the following passage there are blanks, each of which has been numbered. These numbers are printed below the passage and against each, five words are suggested, one of which fits the blank appropriately. Find out the appropriate word in each case.

   It is becoming increasingly fashionable to tax the super-rich. Taxing the super-rich primarily involves (191) the rates of personal income tax or property taxes. It ignores what actually happens with the incomes of the super-rich. Higher expenditure on gods and services results in more revenue being (192)  through indirect taxes. Moreover, if taxes are low, there is no incentive to (193) either income or the acquisition of goods and services. Beyond a point, most income is then invested in financial institutions which, in turn, help fuel industrial and economic growth.

It is axiomatic that a small percentage of the population in any country will always control a major portion of its wealth. The top five per cent will always earn more than fifty per cent of the total income and generate most of nation’s wealth. (194) taxing this class will result in the (195) of a class of the dishonest rich. If a man cannot become rich honestly, he will do so dishonestly. Alternatively, if he has a sense of values and ethics, he has no option but to escape to other countries. The (196) of a large number of India’s brightest engineers and doctors was in no small measure due to our socialist principles and heavy taxes.

The (197) of the tax system is that the honest tax payer is (198)  attacked again and again. The dishonest tax payer (199) pays his tax dues. If the goal is to earn greater revenue, then taxing the super-rich will be counter-productive. The marginal gain in revenue will be temporary and there will be greater loss of revenue through (200) collection of indirect taxes. Hence, it is necessary to re-examine these myriad wasteful welfare schemes.

191. 

(A)  truncating

(B)  increasing

(C)  focusing

(D)  reducing

(E)   channelising

Answer: (B)

192.

(A)  derived

(B)  generated

(C)  validated

(D)  propagated

(E)   lowered

Answer: (B)

193.

(A)  merge

(B)  explore

(C)  violate

(D)  trade

(E)   conceal

Answer: (E)

194.

(A)  Partially

(B)  Incoherently

(C)  Temporarily

(D)  Heavily

(E)   Marginally

Answer: (D)

195.

(A)  defamation

(B)  alienation

(C)  creation

(D)  transition

(E)   suppression

Answer: (C)

196.

(A)  attrition

(B)  termination

(C)  coalition

(D)  migration

(E)   retention

Answer: (D)

197.

(A)  penultimate

(B)  probation

(C)  tragedy

(D)  charge

(E)   incentive

Answer: (C)

198.

(A)  savagely

(B)  earnestly

(C)  hardly

(D)  favourably

(E)   visually

Answer: (A)

199.

(A)  actively

(B)  always

(C)  secretly

(D)  judiciously

(E)   seldom

Answer: (E)

200.

(A)  lower

(B)  alarming

(C)  regular

(D)  volatile

(E)   expected

Answer: (A)

State Bank of India PO Examination 2014 Question Paper With Answer Key

State Bank of India PO Exam-2014
State Bank of India PO Examination 2014 Question Paper With Answer Key

State Bank of India PO Exam-2014

Solved Paper

Test-I

Reasoning

   Directions-(Q. 1-4) Study the following information and answer the question given below-

   J, K, L, M, N, O, P and R are sitting around a circular table facing the centre. Each of them was born in a different year – 1971, 1975, 1979, 1980, 1981, 1984, 1985 and 1990, not necessarily in the same order.

M is sitting second to the right of K. L is sitting third to the right of J. L and J were born before 1980. Only the one born in 1984 is sitting exactly between J and K. N who is the eldest is not an immediate neighbor of J and M. R is older than only M. R is sitting second to the right of P. P is not an immediate neighbor of N. J is  younger than L. K was born before O.

1. Who is the second eldest in the group?

(A)  J

(B)  L

(C)  K

(D)  P

(E)   None of above

Answer: (B)

2. Who is sitting third to the right of O?

(A)  The one born in 1979

(B)  The one born in 1980

(C)  The one born in 1985

(D)  The one born in 1984

(E)   None of these

Answer: (B)

3. In which year was R born?

(A)  1979

(B)  1975

(C)  1980

(D)  1985

(E)   None of these

Answer: (D)

4. Which of the following statements is true regarding K?

(A)  K is sitting fourth to the right of R

(B)  The one who was born in 1975 is to the immediate left of K

(C)  K is younger than R

(D)  There are four people sitting between N and K

(E)   None is rrue

Answer: (E)

Direction-(Q. 5-10) In each of the questions below, three statements, are given followed by conclusions/group of conclusions numbered I and II. You have to assume all the statements to be true even if they seem to be at variance from the commonly known facts and then decide which of the given two conclusions logically follows from the information given in the statements.

Give Answer

(A) If only conclusion I follows

(B) If only conclusion II follows

(C) If either conclusion I or conclusion II follows

(D) If neither conclusion I nor conclusion II follows

(E) If both conclusions I and II follow

For Q. 5 and 6

Statements : All pens are books.

Some books are pages.

All pages are papers.

5. Conclusion : No paper is a pen. II. At least some pages are pens.

Answer: (B)

6. Conclusions : All books are papers. II. Some books are papers.

Answer: (B)

For Q. 7and 8

Statements : Some Ds are Gs.

All Gs are Ks.

All Ks are Ls.

7. Conclusions: At least some Ds are Ls. II. All Gs are Ls.

Answer: (A)

8. Conclusion : At least some Ks are Ds. II. All Ds are Ls.

Answer: (A)

For Q. 9 and 10

Statements : Some files are folders.

All folders are pockets.

No pockets is a bag.

9. Conclusion : All pockets are files. II. All files are bags.

Answer: (D)

10. Conclusions : At least some bags are folders. II. All folders are files.

Answer: (D)

Directions-(Q. 11-14) Following are steps of an input. Rearrange them and answer the questions-

Input

(a) metal   offer  arrange  blue  kite

(b) arrange  blue  kite  metal offer

(c) kite   metal  offer  arrange  blue

(d) offer   arrange   blue  kite  metal

(e) blue  kite  metal  offer   arrange

11. Which of the following is step III?

(A)  (a)

(B)  (b)

(C)  (c)

(D)  (d)

(E)   (e)

Answer: (C)

12. Which of the following is first from the right end of step II?

(A)  arrange

(B)  kite

(C)  metal

(D)  blue

(E)   None of above

Answer: (B)

13. Which of the following is step V?

(A)  (a)

(B)  (b)

(C)  (c)

(D)  (d)

(E)   (e)

Answer: (B)

14. Which of the following is step IV?

(A)  (a)

(B)  (b)

(C)  (c)

(D)  (d)

(E)   (e)

Answer: (E)

Directions-(Q. 15 and 16) Four of the following five are alike in a certain way and so form a group. Which one does not belong to the group?

15.

(A)  Stem

(B)  Branch

(C)  Leaf

(D)  Web

(E)   Fruit

Answer: (D)

16.

(A)  Clone

(B)  Replica

(C)  Duplicate

(D)  Copy

(E)   Fake

Answer: (E)

Directions-(Q. 17-21) Study the following information and answer the questions that follow-

   In a certain code language, ‘hope to see you’ is coded as ‘re so na di’. ‘please come to see the party’ is coded as ‘fi gen a di ke zo’, ‘hope to come’ is coded as ‘di so ge’ and ‘see you the party’ is coded as ‘re fi zo na’.

17. How is ‘please’ coded in the given code language?

(A)  di

(B)  ke

(C)  fi

(D)  na

(E)   None of above

Answer: (B)

18. What does the code ‘so’ stand for in the given code language?

(A)  hope

(B)  come

(C)  see

(D)  to

(E)   None of these

Answer: (A)

19. How is ‘party’ coded in the given code language?

(A)  Either ‘re’ or ‘fi’

(B)  Either ‘zo’ or ‘na’

(C)  Either ‘zo’ or ‘fi’

(D)  Either ‘zo’ or ‘ge’

(E)   Either ‘ke’ or ‘fi’

Answer: (C)

20. How will ‘please see you’ be coded in the given code language?

(A)  re na ke

(B)  so re na

(C)  zo re na

(D)  na di ke

(E)   ke re ge

Answer: (A)

21. Which one of the following will be coded as ‘so di re’ in the given code language?

(A)  you see hope

(B)  hope you please

(C)  hope you come

(D)  the hope to

(E)   you hope to

Answer: (E)

Directions-(Q. 22-25) In these questions, relationship between different elements is shown in the statements. These statements are followed by two conclusions.

Mark answer

(A) If only conclusion I follows

(B) If only conclusion II follows

(C) If either conclusion I or II follows

(D) If neither conclusion I nor II follows

(E) If both conclusions I and II follow

22. Statement : A ≥ B = C; B < D ≤ E

Conclusions : I. D > A II. E > C

Answer: (B)

23. Statement : L > U ≥ K; Z < U < R

(I) L > Z

(II) K < R

Answer: (E)

24. Statement : Y < J = P ≥ R > I

Conclusions : I. J > I

(II) Y < R

Answer: (D)

25. Statement : V ≥ K > M = N; M > S; T < K

Conclusions : I. T < N

  1. V = S

Answer: (D)

Directions- (Q. 26-30) Each of the questions below consists of a question and two statements numbered I and II given below it. You have to decide whether the data given in the statements are sufficient to answer the questions.

   Read both the statements and-

Give Answer

(A) If the data in statement I alone are sufficient to answer the question, while the data in statement II alone are not sufficient to answer the question

(B) If the data in statement II alone are sufficient to answer the question, while the data in statement I alone are not sufficient to answer the question.

(C) If the data either in statement I alone or statement II alone are sufficient to answer the question

(D) If the data given in both statements I and II together are not sufficient to answer the question

(E) If the data in both statements I and II together are necessary to answer the questions

26. How is J related to K ?

(I) J’s father P is the brother of N. N is K’s wife.

(II) J is the son of P. P is the brother of N. N is K’s wife.

Answer: (E)

27. On which floor of the building does G stay.

(The building has five floors 1, 2, 3, 4, 5) ?

(I) Only the even numbered floors are occupied and G does not stay on the second floor.

(II) G does not stay on an odd numbered floor.

Answer: (A)

28. How many days did Raju take to complete his assignment?

(I) Mohit correctly remembers that Raju took more than 3 days but less than 9 days to complete his assignment.

(II) Mina correctly remembers that Raju took more than 7 days but less than 11 days to complete his assignment.

Answer: (E)

29. How is the word ‘GATES’ coded in the code language?

(I) ‘BRICK’ is coded as ‘JDJSC’ and ‘PIN’ is coded as ‘OJQ’.

(II) ‘WATER’ is coded as ‘SFUBX’ and ‘DISH’ is coded as ‘ITJE’.

Answer: (C)

30. Among A, B, C, D which school has the highest number of students?

(I) School A has lesser students than school D.

(II) School C has lesser students than school D.

Answer: (D)

Directions-(Q. 31-35) In these questions, relationship between different elements is shown in the statements. The statements are followed by two conclusions.

Give Answer

(A) If only conclusion I is true

(B) If only conclusion II is true

(C) If only conclusion I or II is true

(D) If only conclusion I nor II is true

(E) If both conclusions I and II are true

31. Statement : T < P ≤ U; L > U ≤ K; P ≥ R

Conclusions : I. K ≥ R II. L > R

Answer: (E)

32. Statement : H = I ≤ R; M ≥ R < S

Conclusions : I. M = I II. M > I

Answer: (C)

33. Statement : D > H ≥ N; S > I ≤ H

Conclusions : I. N ≤ S II. I < D

Answer: (B)

34. Statements : P ≤ O < I; P > Y > W

Conclusions : I. Y ≤ I II. O > W

Answer: (E)

35. Statement : A ≥ B > C ≥ F; Z < C ≤ D < E

Conclusions : (I) A > Z II. F < E

Answer: (D)

Directions- (Q. 36-40) Study the following information to answer the given questions-

   Six people- C, D, E, F, G and H are standing in a straight line facing north not necessarily in the same order. D is standing second to the right of F. C is standing fourth to the left of H and H is not standing on the extreme end of the line. E is standing second to the right of D.

36. What is the position of G with respect to E?

(A)  Immediate left

(B)  Second to the left

(C)  Third to the left

(D)  Third to the right

(E)   None of above

Answer: (C)

37. Which of the following pairs represents the people standing at the extreme ends of the line?

(A)  FH

(B)  CE

(C)  DE

(D)  CH

(E)   None of above

Answer: (B)

38. Who is standing second to the right of C?

(A)  F

(B)  D

(C)  G

(D)  E

(E)   None of above

Answer: (C)

39. Four of the following five are alike in a certain way based on their positions in the above arrangement and so form a group. Which of the following does not belong to the group?

(A)  CG

(B)  GE

(C)  GH

(D)  DE

(E)   FD

Answer: (B)

40. If all the people are asked to stand in an alphabetical order from left to right, the positions of how many will remain unchanged?

(A)  One

(B)  Two

(C)  Three

(D)  None

(E)   None of above

Answer: (D)

Directions-(Q. 41-45) In each question below is given a statement followed by three courses of action lettered (a), (b) and (c). A course of action is a step or administrative decision to be taken for improvement, follow-up or further action in regard to the problem, policy, etc. On the basis of the information given in the statement, you have to Assume everything in the statement to be true, then decide which of the suggested courses of action logically follow(s) for pursuing.

41. Statement :

There has been a continuous increase in the number of dropout of student of Govt. run primary schools in the state.

Courses of action:

(a) Govt. should immediately set up a committee to review the situation and suggest measures to reverse the trend.

(b) Govt. should conduct orientation programmes for parents of the students emphasizing on the need for educating their children.

(c) Govt. should close down such state run primary schools where dropout rates are more than fifty per cent.

(A)  Only (a) follows

(B)  Only (b) follows

(C)  Only (c) follows

(D)  Only (a) and (b) follow

(E)   None of above

Answer: (D)

42. Statement : Every year during monsoon condition, most of the roads in the city deteriorates causing immense problem to the commuters.

Courses of action :

(a) The civic body should include a heavy Penalty clause while awarding future contracts for road repairs.

(b) The civic officials in charge of maintenance of city roads should be asked to explain why the condition of the roads worsens every year.

(c) General public should avoid taking their vehicles out during monsoon.

(A)  Only (a) follows

(B)  Only (b) follows

(C)  Only (a) and (b) follow

(D)  Only (b) and (c) follow

(E)   None of above

Answer: (C)

43. Statement : During the past few days more and more number of indoor patients of the local Govt. hospital are diagnosed to be suffering from malaria.

Course of action:

(a) All such patients who are suffering from malaria should immediately be discharged from the hospital.

(b) The hospital authority should immediately put a ban on admitting new patients into the hospital.

(c) All such patients who are suffering from malaria should be kept in an isolated ward.

(A)  Only (a) follows

(B)  Only (b) follows

(C)  Only (c) follows

(D)  None follows

(E)   All follows

Answer: (D)

44. Statement : Many public sector undertakings have been making losses for the past few years and the situation is equally bad in the current year.

Course of action:

(a) These loss making public sector companies should immediately be closed down.

(b) The Govt. should scout for potential buyers in the private sector to sell these companies to get back part of the investments made by the Govt.

(c) All the employees of these companies should be retrenched with adequate compensation and the fixed assets may be put up for sale.

(A)  None follows

(B)  Only (a) and (b) follow

(C)  Only (b) and (c) follow

(D)  All (a), (b) and (c) follow

(E)   None of these

Answer: (A)

45. Statement : Many shops in the local market have extended their shops and occupied most part of the footpath in front of their shops.

Course of action:

(a) The civic authority should immediately activate a task force to clear all the footpath encroached b the shop owners.

(b) The civic authority should charge hefty penalty to the shop owners for occupying footpath.

(c) The civic authority should set up a monitoring system so that such encroachments do not recur in future.

(A)  None follows

(B)  Only (a) and (b) follow

(C)  Only (b) and (c) follow

(D)  All (a), (b) and (c) follow

(E)   None of these

Answer: (D)

46. In which of the following expressions will the expression ‘P > F’ be definitely false?

(A)  F = B > P ≤ M

(B)  P > B ≥ M = F

(C)  P ≥ B < F ≤ M

(D)  B < P ≤ M < F

(E)   None of above

Answer: (A)

Directions- (Q. 47 and 48) Read the following information carefully and answer the questions which follow-

   If ‘A × B’ means A is the son of B.

If ‘A + B’ means A is the father of B.

If  ‘A > B’ means A is the daughter of B.

If ‘A < B’ means A is the wife of B.

47. Which of the following pairs of people represent first cousins with regard to the relations given in the expressions, if it is provided that A is the sister of

J : ‘L > V < J + P’ ‘S × A < D + F < E + K’

(A)  LP

(B)  SP

(C)  SK

(D)  SF

(E)   Cannot be determined

Answer: (E)

48. What will come in the place of the question mark, if it provided that M is the grandmother of F in the expression : ‘F × R < S ? M’

(A)  >

(B)  <

(C)  +

(D)  ×

(E)   Cannot be determined

Answer: (D)

49. In which of the following expressions will the expression P < M be definitely true?

(A)  M < R > P ≥ S

(B)  M ≥ S = P < F

(C)  Q < M < F = P

(D)  P = A < R < M

(E)   None of above

Answer: (D)

50. In a class of 42 children, Joseph’s rank is sixteenth from the top. Kevin is seven ranks below Joseph. What is Kevin’s rank from the bottom?

(A)  22nd

(B)  20th

(C)  19th

(D)  23rd

(E)   25th

Answer: (B)

Test-II

Data Analysis & Interpretation

   Directions-(Q. 51-56) Study the following graph carefully to answer the given question-

51. If the number of students with Mechanical specialization in each institute increased by 20% and the number of students with Electronics specialization in each institute deceased by 15% from 2012 to 2013, total number of students with Mechanical from all the institutes in 2013 is approximately, what per cent of the total number of students with Electronics specialization from all the institutes in 2013?

(A)  122

(B)  116

(C)  162

(D)  132

(E)   178

Answer: (E)

52. What is the ratio between total number of students in institutes R and V respectively?

(A)  39 : 43

(B)  39 : 44

(C)  37 : 44

(D)  39 : 45

(E)   38 : 43

Answer: (B)

53. What is the difference between total number of students with IT specialization from all the institutes and the total number of students with Mechanical specialization from all the institutes together?

(A)  260

(B)  240

(C)  280

(D)  220

(E)   250

Answer: (D)

54. If the number of students in institutes P, Q and R with IT specialization increased by 15%, 22% and 10% respectively from 2012 to 2013, what was the total number of students with IT specialization in the three institutes together in 2013?

(A)  1028

(B)  1056

(C)  1043

(D)  1142

(E)   1145

Answer: (C)

55. If out of the total number of students for all three specialization together in institute Q number of students having liking for Music, Painting and Cricket are in the ratio of 5 : 6 : 7 respectively, then what is the number of students liking Music from this institute?

(A)  250

(B)  300

(C)  350

(D)  360

(E)   280

Answer: (A)

56. In institutes P, T and U the percentage of girls out of total number of students with Electronics specialization is respective institute is 50%, 55% and 48% respectively, what is the total number boys in these three institutes with Electronics specialization?

(A)  340

(B)  386

(C)  356

(D)  360

(E)   314

Answer: (A)

Directions-(Q. 57-62) Study the table and answer the given question-

57. Which of the given statements is/are true?

(a) Total advertisement revenue generated from online version by magazine T in all the given months together is exactly 44% less than the total advertisement revenue generated from printed version by the same magazine in all the given months together.

(b) The difference between advertisement revenue generated (from both online and printed version) by all the given magazines in January and advertisement revenue generated (from both online and printed version) by all the given magazines in June is Rs 62,000.

(c) Only for one magazine advertisement revenue generated from printed version displayed a constant increase from previous months during the given 6 months period.

(A)  Only (b) and (c)

(B)  Only (a) and (b)

(C)  Only (a)

(D)  Only (b)

(E)   All (a), (b) and (c)

Answer: (D)

58. Total advertisement revenue generated from online version by Magazine U in all the given months together is by what per cent more than the total advertisement revenue generated from printed version by the same magazine in all the given months together?

(A)  25

(B)  27.5

(C)  35

(D)  30

(E)   32.5

Answer: (C)

59. In case of Magazine R, between which two given months was there approximately 15% rise in advertisement revenues generated (from both printed and online version together)?

(A)  January-February

(B)  February-March

(C)  May-June

(D)  March-April

(E)   April-May

Answer: (D)

60. The respective ratio of advertisement revenue generated from printed version by Magazine P to advertisement revenue generated from online version by the same magazine in July is same as the respective ratio of advertisement revenue generated from printed version by Magazine Q to advertisement revenue generated from online version by the same magazine in March. If the advertisement revenue generated from online version by Magazine P in July was Rs 1,08,000, what was the advertisement revenue generated from printed version by the same magazine in July?

(A)  Rs 1,87,000

(B)  Rs 1,53,000

(C)  Rs 1,36,000

(D)  Rs 1,70,000

(E)   Rs 1,19,000

Answer: (B)

61. What is the respective ratio between total advertisement revenue generated from online version of all the given magazines together in February and total advertisement revenue generated from printed version of all the given magazines together in May?

(A)  33 : 28

(B)  39 : 28

(C)  27 : 16

(D)  33 : 20

(E)   27 : 22

Answer: (C)

62. Total advertisement revenue generated from printed version by Magazine P in January and April together is what per cent less than the total advertisement revenues generated from online version by Magazine S in February and June together?

(A)  22

(B)  14

(C)  27

(D)  19

(E)   17

Answer: (E)

63. If the number of male Post-graduate employees in Country H is 1800, what per cent of female employees in that particular country is Post-graduate?

(A)  76

(B)  74

(C)  72

(D)  64

(E)   68

Answer: (A)

64. In which of the given countries is percentage of women employees to number of employees (both male and female) in that country the second lowest?

(A)  G

(B)  B

(C)  E

(D)  H

(E)   D

Answer: (B)

65. What is the respective ratio between total number of male employees in countries B and H together and total number of female employees in countries C and D together?

(A)  63 : 52

(B)  51 : 38

(C)  77 : 64

(D)  69 : 44

(E)   57 : 40

Answer: (D)

66. What is the difference between average number of Post-graduate employees in countries A, B and D together and average number of Post-graduate employees in countries F, G and H together?

(A)  282

(B)  276

(C)  316

(D)  342

(E)   294

Answer: (E)

67. Which of the given countries has the highest number of average employees per office?

(A)  F

(B)  H

(C)  B

(D)  C

(E)   D

Answer: (D)

Directions-(Q. 68-72) Study the following graph carefully to answer the question-

68. Total number of students playing Carrom and Hockey together from school P is what per cent of the total number of students playing these two games together from school R?

(A) 

(B) 

(C)  72

(D) 

(E) 

Answer: (C)

69. If the number of students playing each game in school S is increased by 15% and the number of students playing each game in school Q is decreased by 5% what will be the difference between number of students in schools S and Q?

(A)  54

(B)  218

(C)  356

(D)  224

(E)   205

Answer: (E)

70. If out of the students playing Cricket from schools Q, S and T 40%, 35% and 45% respectively got selected for state level competition, what was the total number of students got selected for State level competition from these three schools together?

(A)  346

(B)  241

(C)  292

(D)  283

(E)   268

Answer: (D)

71. Total number of students playing Hockey from all schools together is approximately what per cent of the total number of students playing Cricket from all school together?

(A)  84

(B)  74

(C)  72

(D)  78

(E)   70

Answer: (D)

72. From school P, out of the studnts playing Carrom, 40% got selected for State level competition, out of which 25% further got selected for National level competition. From school T, out of the students playing Carrom, 45% got selected for State level competition, out of which two-third further got selected for National level competition. What is the total number of students playing carom from these two schools who got selected for National level competition?

(A)  106

(B)  98

(C)  112

(D)  108

(E)   96

Answer: (A)

Directions-(Q. 73-78) Study the following table carefully and answer the question-

73. Urban population of Maharashtra and Orissa together is what per cent of the total population of these two states ? (rounded off to two digits after decimal?

(A)  32.49

(B)  35.59

(C)  38.55

(D)  32.85

(E)   36.57

Answer: (B)

74. Approximately, by what per cent is the urban population of Maharashtra less than its rural population?

(A)  33

(B)  39

(C)  49

(D)  45

(E)   34

Answer: (B)

75. What per cent of the total population of West Bengal, Orissa and Madhya Pradesh together is illiterate ? (rounded off to nearest integer)

(A)  28

(B)  34

(C)  29

(D)  33

(E)   31

Answer: (E)

76. What is the difference between number of graduates from Madhya Pradesh and Uttaranchal?

(A)  3,66,340

(B)  3,49,185

(C)  2,88,185

(D)  3,82,340

(E)   3,55,840

Answer: (B)

77. What is the total number of graduates from Orissa, West Bengal and Maharashtra together?

(A)  15,09,695

(B)  15,29,559

(C)  16,43,780

(D)  16,19,455

(E)   16,29,095

Answer: (C)

78. What is the ratio between total female population of Tamil Nadu and Jharkhand together and the total male population of these two states together respectively?

(A)  697 : 798

(B)  715 : 797

(C)  708 : 797

(D)  698 : 797

(E)   696 : 797

Answer: (C)

Directions-(Q. 79-85) Study type following information carefully to answer the question-

   In Medical College there are 1600 students studying Dentistry  and Homeopathy. Each student from each course knows one or more languages out of English, Hindi and Bangla 45% of the students study Dentistry and remaining students study Homeopathy.

Out of the students studying Dentistry boys and girls are in the ratio of 5 : 3 respectively.

Out of the boys studying Dentistry, 16% know only English, 10% know only Hindi and 4% know only Bangla, 24% know English as well as Hindi, 20% know English as well as Bangla, 14% know Hindi as well as Bangla Remaining boys know all three languages.

Out of the girls studying Dentistry, 20% know only English, 10% know only Hindi and 10% know only Bangla 20% know English as well as Hindi, 20% know English as well as Bangla, 10% know Hindi as well as Bangla. Remaining girls know all three languages.

Out of the students studying Homeopathy boys and girls are in the ratio of 4 : 7 respectively.

Out of the boys studying Homeopathy, 20% know only English, 15% know only Hindi and 5% know only Bangla, 15% know English as well as Hindi, 25% know English as well as Bangla, 10% know Hindi as well as Bangla. Remaining boys know all three languages.

Out of the girls studying Homeopathy, 15% know only English, 15% know only Hindi and 5% know only Bangla, 20% know English as well as Hindi, 20% know English as well as Bangla 15% know Hindi as well as Bangla. Remaining girls know all three languages.

79. How many students studying Dentistry know only either English or Hindi?

(A)  166

(B)  162

(C)  308

(D)  198

(E)   248

Answer: (D)

80. Total how many students in the college know all three languages?

(A)  108

(B)  132

(C)  169

(D)  137

(E)   142

Answer: (C)

81. What per cent of the total number of girls in the college know Bangla?

(A)  45

(B)  40

(C)  48

(D)  42

(E)   50

Answer: (E)

82. How many students studying Homeopathy, do not know English?

(A)  292

(B)  232

(C)  228

(D)  298

(E)   207

Answer: (A)

83. Out of the students studying Homeopathy what is the ratio between number of boys knowing English and the number of girls knowing Hindi respectively?

(A)  3 : 5

(B)  2 : 3

(C)  9 : 11

(D)  9 : 13

(E)   1 : 3

Answer: (B)

84. Out of the total number of students studying Dentistry, what per cent knows at least two languages?

(A) 

(B) 

(C) 

(D) 

(E) 

Answer: (D)

85. What per cent of the total number of girls in the college do not know Hindi? (rounded off to nearest integer)

(A)  38

(B)  46

(C)  48

(D)  36

(E)   43

Answer: (E)

Directions-(Q. 86-93) Refer to the pie chart and answer the given question-

86. Total number of luxury watches sold by stores C and H together is by what per cent less than total number of watches (sport watches and luxury watches) sold by stores F and H together?

(A) 

(B) 

(C) 

(D) 

(E) 

Answer: (D)

87. Number of watches (sport watches and luxury watches) sold by store D is by what per cent more than total number of sport watches sold by stores C, F and G together?

(A)  22.5

(B)  25

(C)  27.5

(D)  35

(E)   30

Answer: (B)

88. What is the respective ratio between total number of sport watches sold by stores A and B together and total number of watches (sport and luxury) sold by stores C and F together?

(A)  64 : 77

(B)  48 : 61

(C)  56 : 77

(D)  64 : 81

(E)   48 : 73

Answer: (A)

89. What is the average number of luxury watches sold by stores A, D, E, F and H together?

(A)  3152

(B)  3296

(C)  3548

(D)  3186

(E)   3428

Answer: (B)

90. If the number of watches sold by stores A, D and E increased by 10%, 35% and 15% respectively from 2004 to 2005 what was the total number of luxury watches sold by these three stores in 2005?

(A)  16172

(B)  14966

(C)  15848

(D)  15964

(E)   16392

Answer: (D)

91. What is the central angle corresponding to the number of watches (sport watches and luxury watches) sold by store B?

(A)  39.6°

(B)  36.6°

(C)  44.2°

(D)  42.2°

(E)   45.4°

Answer: (A)

92. Total number of sport watches sold by stores E and F together is what per cent of the number of watches (sport watches and luxury watches) sold by store E?

(A) 

(B) 

(C) 

(D) 

(E) 

Answer: (E)

93. What is the difference between average number of sport watches sold by stores B, C, G and H together and average number of luxury watches sold by the same stores together?

(A)  1100

(B)  1200

(C)  900

(D)  800

(E)   1300

Answer: (A)

Directions-(Q. 94-100) Refer to the graphs and answer the given question:

94. Scheme M offers Simple Interest at a certain rate of Interest (p.c.p.a.). If the difference between the interest caused by gautam and Rudra from scheme M after 4 years is Rs 44365.20 what is the rate of interest (p.c.p.a)?

(A)  17.5

(B)  18

(C)  16.5

(D)  20

(E)   15

Answer: (C)

95. What is respective ratio between total amount invested by Gautam in schemes O and Q together total amount invested by Rudra in the same schemes together?

(A)  31 : 44

(B)  31 : 42

(C)  27 : 44

(D)  35 : 48

(E)   29 : 38

Answer: (A)

96. If scheme P offers compound interest (compounded half yearly) @ 16 p.c.p.a. what would be sum of interest caused by Gautam and Rudra from scheme P after one year.

(A)  Rs 10,244

(B)  Rs 10,464

(C)  Rs 9,872

(D)  Rs 9,984

(E)   Rs 9,442

Answer: (D)

97. If scheme O offers compound interest (compounded annually) @ 12 p.c.p.a. what is the difference between interest earned by Gautam and Rudra from scheme O after 2 years?

(A)  Rs 1628.16

(B)  Rs 1584.38

(C)  Rs 1672.74

(D)  Rs 1536.58

(E)   Rs 1722.96

Answer: (A)

98. What is the average amount invested by Gautam in schemes M, N, O, P and Q together?

(A)  Rs 29,248

(B)  Rs 30,562

(C)  Rs 31,126

(D)  Rs 29,688

(E)   Rs 28,848

Answer: (A)

99. Rudra invested in scheme R for 4 If scheme R offers simple interest @ 7 p.c.p.a. for the 1st two years and then compound interest @ 10 p.c.p.a. (compounded annually) for the 3rd and 4th year what will be the interest carried by Rudra after 4 years?

(A)  13548

(B)  12096

(C)  12242

(D)  12364

(E)   11886

Answer: (B)

100. If amount Invested by Gautam in scheme S is equal to the amount invested by him in scheme N. The rate of interest (p.c.p.a.) of scheme S and N are same. The only difference in scheme S offers compound interest ( compounded annually) whereas the scheme N offers simple interest. If the difference between the interest caused by Gautam from both the schemes after 2 years is Rs 349.92 what is the rate of interest?

(A)  9

(B)  5

(C)  13

(D)  11

(E)   7

Answer: (A)

Test-III

General Awareness, Marketing & Computer

101. A ‘Basic Savings Bank Deposit Account’ facility is available-

(A)  to all individuals irrespective of their background

(B)  all individuals from below poverty line families only

(C)  only to individuals from the SC/ST only

(D)  only to individuals from the minority communities only

(E)   only to individuals from weaker sections of the society

Answer: (A)

102. Which of the following is an example of an input device?

(A)  Monitor

(B)  Scanner

(C)  Printer

(D)  CD

(E)   Speaker

Answer: (B)

103. The Reserve Bank of India has been critical of home loan products with comparatively low interest rates in the initial years, but higher in the subsequent years which are popularly known as the-

(A)  Teaser Rates

(B)  Cheater Rates

(C)  Twister Rates

(D)  Cheaper Rates

(E)   Trickster Rates

Answer: (A)

104. Relationship marketing is also known as-

(A)  Loyalty marketing

(B)  Experiential marketing

(C)  Value marketing

(D)  Promotional Marketing

(E)   Brand marketing

Answer: (A)

105. PCs are considered fourth-generation and contain-

(A)  information

(B)  data

(C)  vacuum tubes

(D)  microprocessors

(E)   transistors

Answer: (D)

106. Which of the following is not an aggressiveness strategy?

(A)  All the given options are aggressiveness strategies

(B)  Building

(C)  Harvesting

(D)  Holding

(E)   Intensification

Answer: (E)

107. Which of the following nations is not a member of SAARC?

(A)  Nepal

(B)  Mauritius

(C)  Bhutan

(D)  Afghanistan

(E)   Maldives

Answer: (B)

108. Market price is also known as-

(A)  List price

(B)  Value price

(C)  Effective price

(D)  Retail price

(E)   Wholesaler price

Answer: (C)

109. The phenomenon, when a customer dislikes a product and talks against the product-

(A)  Misinformation

(B)  Unfavourable environment

(C)  Propaganda

(D)  Bad impression

(E)   Bad mouth

Answer: (E)

110. The RBI policy rate which is purely an indicative rate used by the Reserve Bank of India to signal long-term outlook on interest rates is-

(A)  Bank Rate

(B)  Repo Rate

(C)  Call Money Rate

(D)  Notice Money Rate

(E)   Reverse Repo Rate

Answer: (A)

111. The amount specified as the Cash Reserve Ratio (CRR) is held in cash and cash equivalents and is stored in bank vaults or parked with-

(A)  Small Industries Development Bank of India (SIDBI)

(B)  Government of India (GOI)

(C)  Reserve Bank of India (RBI)

(D)  State Bank of India (SBI)

(E)   Rural Infrastructure Development Fund (RIDF)

Answer: (C)

112. As consumer banking frauds are on the rise, a Reserve Bank of India (RBI) group has suggested the use of PKI in order to payment system in the country. The abbreviation ‘PKI’ stands for-

(A)  Public Key Infrastructure

(B)  Personal Key Infrastructure

(C)  Private Key Infrastructure

(D)  Permanent Key Infrastructure

(E)   Proprietary Key Infrastructure

Answer: (A)

113. The Reserve Bank of India recently issued guidelines allowing minors over 10 years of age to operate bank accounts independently with a view to_

(A)  Promote Financial Inclusion

(B)  Increase low cost deposits of banks

(C)  Improve CASA percentage of banks

(D)  Mobilize Savings Bank Deposits of banks

(E)   To put ATMs and other infrastructure to optimum use

Answer: (A)

114. Java is referred to as a(an)

(A)  high-level language

(B)  complex language

(C)  hardware device driver

(D)  low-level language

(E)   programming mid-level language

Answer: (A)

115. Kepler 78b is the name off-

(A)  an American nuclear missile

(B)  a Russian armored vehicle

(C)  a precision measuring instrument

(D)  a Spanish Sea Vessel

(E)   an earth-sized planet far beyond our Solar system

Answer: (E)

116. The Depositor Education and Awareness Fund (DEAF) has been set up with-

(A)  Public Sector Banks

(B)  Indian Banks Association

(C)  State Bank of India

(D)  Reserve Bank of India

(E)   Government of India

Answer: (A)

117. The operating system is the most common type of ………. software.

(A)  application

(B)  antivirus

(C)  communication

(D)  system software

(E)   word-processing software

Answer: (D)

118. Hindustan Motors, India’s oldest car maker, recently shut down its factory at Uttarpara in West Bengal and suspended the production of the iconic-

(A)  Utility Vehicle Pushpak

(B)  Ambassador Car

(C)  Bedford Truck

(D)  Contessa Car

(E)   Morris Oxford Car

Answer: (B)

119. In Trade Finance, a financial transaction involving the purchase of receivables from exporters by a third party who takes all the risks associated with the receivables is known as-

(A)  Forfeiting

(B)  Securitisation

(C)  Negotiation

(D)  Factoring

(E)   Assignment

Answer: (D)

120. Which is not a storage device?

(A)  Floppy disk

(B)  Printer

(C)  DVD

(D)  Hard disk

(E)   CD

Answer: (B)

121. ………. is a written description of a computer program’s functions.

(A)  Explanatory instructions

(B)  Graphical user interface

(C)  Plug and play

(D)  README files

(E)   Documentation

Answer: (D)

122. The term ‘pre-shipment finance’ relates to-

(A)  Export Credit

(B)  Farm Credit

(C)  Consumer Credit

(D)  Investment Credit

(E)   Industrial Credit

Answer: (A)

123. The Government of India has undertaken a programme of recapitalization of Public Sector Banks to help them enhance business growth and ………..

(A)  Capital Adequacy Norms

(B)  Ratio of Non Performing Assets

(C)  Per Employee Business Ratio

(D)  CASA Ratio

(E)   Credit to Deposit Ratio

Answer: (A)

124. Which of the following key is used to delete characters to the left of the cursor?

(A)  Alt = Delete

(B)  Shift

(C)  Esc

(D)  Delete

(E)   Backspace

Answer: (E)

125. Senior Supreme Court advocate Mr. Mukul Rohatgi has recently been appointed as the new-

(A)  Principal Secretary to the PM

(B)  Director of Public Prosecutions

(C)  Advocate General of India

(D)  Solicitor General of India

(E)   Attorney-General of India

Answer: (E)

126. Products that are usually purchased due to adversity and high promotional back up rather than desire are called-

(A)  Regular goods

(B)  Unsought goods

(C)  Preferred goods

(D)  Sought goods

(E)   Unique goods

Answer: (B)

127. The amount of memory (RAM or ROM) is measured in-

(A)  Bytes

(B)  Bits

(C)  Mega Bytes

(D)  Mega Bits

(E)   Hertz

Answer: (A)

128. Who amongst the following has recently been sworn in as the first female judge at Pakistan’s National Sharia Cout, which hears cases under the Islamic legislation?

(A)  Ashraf Jilani

(B)  Ashraf Jabber

(C)  Ashraf Jana

(D)  Ashraf Jahangir

(E)   Ashraff Jehan

Answer: (E)

129. The Canadian Government has announced the suspension of funding to the Commonwealth Secretariat for two years over the issue of human rights abuse by-

(A)  United Kingdom

(B)  New Zealand

(C)  South Africa

(D)  Australia

(E)   Sri Lanka

Answer: (E)

130. Nobel laureate and recipient of the US Presidential Medal and also the US congressional medal Mr. Muhammad Yunus is well known for his contribution to Bangladesh’s

(A)  Consumer Credit Sector

(B)  Agro Credit Sector

(C)  Housing Finance Sector

(D)  Micro-Credit Sector

(E)   Retail Credit Sector

Answer: (D)

131. Which of the following is not a technique of measuring customer satisfaction and monitoring customer’s complaint?

(A)  Exit interviews

(B)  Web information hotlines

(C)  Business analysis

(D)  Customer complaints database

(E)   Telephone information hotlines

Answer: (C)

132. Who amongst the following has been named as the goodwill ambassador for the 35th National Games to be held in Kerala in early 2015?

(A)  S. Sreesanth

(B)  Sachin Tendulkar

(C)  Mary Kom

(D)  P. T. Usha

(E)   Mahendra Singh Dhoni

Answer: (B)

133. The official football of the Brazil 2014 FIFA World Cup is named as ‘Brazuca’ which means-

(A)  The spirit of sportsmanship

(B)  The spirit of the game is in playing

(C)  The Brazilian way of life

(D)  May the best team win

(E)   We play to win

Answer: (C)

134. Underpriced products sell very well, but they produce less revenue than they would have if prices were raised to the-

(A)  variable level

(B)  demand curve level

(C)  Price-floor level

(D)  perceived value level

(E)   value-based level

Answer: (D)

135. A customer requirement about any product in which he/she needs to avail core features in that product is called-

(A)  Read need

(B)  No need

(C)  Stated need

(D)  Unstated need

(E)   Delighted need

Answer: (C)

136. Which of the following is mode for creating charge of Life Insurance Policies?

(A)  Pledge

(B)  Lien

(C)  Assignment

(D)  Hypothecation

(E)   Equitable Mortgage

Answer: (C)

137. Bankers generally do not allow opening of accounts in the name of-

(A)  Executors and Trustees

(B)  Persons of unsound mind

(C)  Illiterate Persons

(D)  Pardanshin ladies

(E)   Visually Impaired Persons

Answer: (B)

138. The hard drive is normally located-

(A)  next to the printer

(B)  plugged into the back of the computer

(C)  underneath the monitor

(D)  on top of the CD-ROM

(E)   inside the system base unit

Answer: (E)

139. The winner of the 51st Femina Miss India 2014 pageant is-

(A)  Jhatalekha Malhotra

(B)  Navneet Kaur Dhillon

(C)  Koyal Rana

(D)  Gail Nicole Da Silva

(E)   Megan Young

Answer: (C)

140. Virtual memory allocates hard disk space to supplement the immediate, functional memory capacity of what?

(A)  ROM

(B)  EPROM

(C)  The registers

(D)  Extended memory

(E)   RAM

Answer: (E)

141. 3/10 net 30 is an example of which of the following?

(A)  Cash discount

(B)  Quantity discount

(C)  Seasonal discount

(D)  Bulk discount

(E)   Trade discount

Answer: (A)

142. Which of the following refers to restarting the system when it is already powered on?

(A)  A strong boot

(B)  Hibernation

(C)  A cold boot

(D)  Standby mode

(E)   A warm boot

Answer: (E)

143. ……… is having more memory addresses than are physically available.

(A)  Virtual memory

(B)  System software

(C)  Applications software

(D)  RAM

(E)   Vertical Memory

Answer: (A)

144. Which of the following refers to dangerous programs that can be ‘caught’ by opening e-mail attachments and downloading software from the internet?

(A)  Utility

(B)  Virus

(C)  Honeypot

(D)  Spam

(E)   App

Answer: (B)

145. Commercial Paper (CP) is an unsecured money market instrument issued in the form of a promissory note Commercial Paper can be issued in denominations of-

(A)  Rs 1 lac or multiples thereof

(B)  Rs 2 lac or multiples thereof

(C)  Rs 3 lac or multiples thereof

(D)  Rs 5 lac or multiples thereof

(E)   Rs 10 lac or multiples thereof

Answer: (D)

146. A company is providing warehousing facility to its channel members. The company is using which of the following?

(A)  Seasonal discount

(B)  Cash discount

(C)  Quantity discount

(D)  Trade discount

(E)   Quality discount

Answer: (D)

147. Aggressive pricing is associated with which of the following stage of product life cycle?

(A)  Not Associated

(B)  Introduction

(C)  Growth

(D)  Maturity

(E)   Decline

Answer: (D)

148. The United Nations (U.N.) and the International Olympic Committee (IOC) have recently signed a historic agreement to use the power of sports to promote-

(A)  Literacy and Awareness

(B)  Peace and economic development

(C)  World amity and friendship

(D)  Peaceful use of nuclear technology

(E)   World trade and commerce

Answer: (B)

149. Which of the following is not an objective of discounts?

(A)  All the given options are objectives of discounts

(B)  Reward competitors

(C)  Reward valuable customers

(D)  Move out-of-date stock

(E)   Increase short-term sales

Answer: (B)

150. The abbreviation ASBA stands for-

(A)  Applications Supported by Blocked Account

(B)  Applications Supported by Bank Account

(C)  Applications Sustained by Blocked Amount

(D)  Applications Services by Blocked Account

(E)   Applications Supported by Blocked Amount

Answer: (A)

Test-IV

English Language

   Directions-(Q. 151-1600 Read the following passage carefully and answer the given question. Certain words have been given in bold to  help you locate them while answering some of the questions.

   On attending a conference, which focused on the role of the services sector in Indian economy I was amazed. The conference gave a very interesting perspective on the roe of the service sector in the growth of the Indian economy in relation to growth rates in agriculture and industry. The current situation in India is that the growth rate of services has overtaken both agriculture and industry and is now contributing to more than fifty per cent of GDP. The services sector has the highest growth rate and is the least volatile sector. Growth is particularly marked in public services. IT and financial services. In some areas the growth rate of the services sector is forty to fifty per cent due to increased use of mobile technologies. India therefore has a services-oriented economy. It hasn’t followed traditional growth models as in China. However, in the process of doing so it has skipped the manufacturing stage and has jumped straight from the agricultural stage to service stage which is also the main reason for the expansion of the service sector. In fact the situation now is such that the growth in the service sector can and will support the growth in the agricultural and industrial sectors. However, the only setback for Indian economy. Is the lack of growth in the manufacturing sector, which causes dependence on other countries which is not so desirable in terms of job creation and increased prosperity.

Population is also a major concern of the Indian economy. As the population of India grows so also does the number of dependents in the population in both the lower and higher age groups. In such a scenario of increasing population, especially in an economy which is still recovering from crisis growth becomes difficult. For such an economy to grow it has to invest. Currently the public sector invests more than it saves. The household sector saves in surplus, but it is not increasing so it cannot continue to support private and public sectors. There is a massive need to spend on agriculture and infrastructure development of the country. Apart from that health and education should also be the priority of the government, particularly the education of women in order to reduce the birth rate.

However, all said and done we cannot deny the fact that growing population of the country can also benefit the economy if considered as a rescind used efficiency. In fact, it is said that in the next two decades a growth window in India is open, which may not come again because the working population to total population ratio rises up to mid 2030’s only. It is important for India to maximize its economic growth in this period for doing so, it will be important for India to absorb the growing labour force. This would and can that most people in the country would be employed (with a steady income), the number of dependents in the population would reduce and with effect the economy would prosper. Absorbing the labour force is also very important if the service sector is to play key role in the growth of Indian economy.

Today to address the issue of poverty in India, there is a need to change the bad sectors to good sectors and in turn to move people from unemployment to employment. Only the service sector can help in doing so and thus can have major impact on poverty. Although the service intensive sectors such as hotels, restaurants, IT are blooming with the growth in human skills there are geographical labour unions and human skills restaurants on labour movement. The key question here, I suppose that-can service sectors lead the economy? For example, services as IT be taken to rural areas? Experts in the conference have suggested that it seems that services could lead the economy, however, there are certain prerequisites for the same. In other words, there needs to be greater equality between the states and a better gender balance. There is also the need for additional fiscal capacity tax reform to fund education, reduction in government debt and revenue account must be kept in balance. Progress is good but still the initial conditions for growth have not yet been achieved.

151. Choose the word which is most opposite in meaning as the word given in bold as used in the passage.

Volatile

(A)  Erratic

(B)  Impatient

(C)  Stable

(D)  Solid

(E)   Strained

Answer: (C)

152. According to the passage, which of the following is/are true about the impact of increasing population on Indian economy?

(a) If India attempts to absorb all the labour force, it will impact the growth of service sector negatively and in turn hamper the economic growth of the country.

(b) As the population of the country increases, the number of dependants in the country also increase which in turn increases the pressure on the economy.

(c)  An increasing population can never lead the economy of the country towards prosperity in fact it can only put strain on the economy of a country.

(A)  All (a), (b) and (c)

(B)  Only (c)

(C)  Both (b) and (c)

(D)  Both (a) and (c)

(E)   Only (b)

Answer: (C)

153. According to the passage, which of the following can be said about the agriculture and industry sectors in India?

(a) Looking at the growth of the service sector in India it can be safely said that the service sector will soon be in a position to support both agriculture and industry sector.

(b) The agriculture and the industrial sector of the country have reached their threshold and there would be no use of further investment in these sectors.

(c) Currently the agriculture and industry sectors contribute lesser as compared to service sector to the GDP of the country.

(A)  Only (a)

(B)  Only (c)

(C)  Both (a) and (c)

(D)  Both (b) and (c)

(E)   All (a), (b) and (c)

Answer: (C)

154. What does the author mean by the statement, “…a ‘growth window’ for India will open”?

(A)  In the coming years the Indian economy will grow as the service sector of the country would be booming due to the focus of the government in that sector.

(B)  In the next two decades Indian economy will have an opportunity to grow as the working population of India will be high as compared to the dependent population.

(C)  There would be only a small period of time in which the economy of India has to grow and if it fails to do so it will never be able to recover from the economic downfall.

(D)  Only upto the next two decades would Indian people be interested in finding jobs in the country beyond which they would search for jobs abroad thus hampering the growth of Indian economy.

(E)   The growth of Indian economy is like a small window and in order to substantiate the growth it is important to increase the size of the window by improving trade relations with other countries.

Answer: (B)

155. Choose the word which is most similar in meaning to the word given in bold as used in the passage.

Prerequisites

(A)  Requirements

(B)  Instincts

(C)  Acknowledgements

(D)  Prohibitions

(E)   Problems

Answer: (A)

156. What does the author mean by the statement, “to change the bad sectors to good sectors.”?

(A)  In order to deal with the problem of poverty it is important to distinguish between good and bad sectors and encourage people to start searching for jobs in the good sectors.

(B)  The government should make efforts to improve the sectors that are not functioning well in order create greater

(C)  Poverty alleviation is possible only if the government understands the importance of good sectors and provides it with necessary opportunities.

(D)  The bad sectors of the country should be identified and such jobs should be outsourced to other underprivileged countries in order to eradicate poverty from those countries.

(E)   People should be educated to ensure that they work only in the good sectors and in turn in the issue of poverty in the country is resolved.

Answer: (B)

157. Choose the word which is most similar in meaning to the word given in bold as used in the passage.

Perspective

(A)  Viewpoint

(B)  Prospect

(C)  Attitude

(D)  Agreement

(E)   Proportion

Answer: (A)

158. Which of the following is True as per the passage?

(A)  India has not followed the conventional model of growth and has moved directly from the agriculture sector to the service sector.

(B)  The service sector of the country is yet to make a mark on the IT and financial sectors of the country.

(C)  With availability of labour and growth in human skills, the service sector of Indian economy is booming limitlessly as there is not restriction on movement of labour.

(D)  India has become self reliant and does not have to depend on other countries because of the development in the manufacturing sector.

(E)   All the given statements are true.

Answer: (A)

159. According to the author, which of following can be said about the growth of service sector in India?

(A)  India is essentially an agrarian economy and is not yet ready to shift focus from agriculture sector to service sector.

(B)  The expansion of service sector in India was see mainly because of the growth in the agricultural sector.

(C)  Indian economy cannot run only on the basis of service sector i.e. without further development in the agriculture and the industrial sector.

(D)  The growth of service sector in India is facilitate by the improvement in technology.

(E)   It will take the service sector at least around two decades before it can over take the agriculture and industry sector.

Answer: (C)

160. Choose the word which is most opposite in meaning to the word given in bold as used in the passage.

Massive

(A)  Prominent

(B)  Proactive

(C)  Short

(D)  Little

(E)   Vast

Answer: (D)

Directions-(Q. 161-170) Read the following passage carefully and answer the given questions. Certain words/phrases are given in bold to help you to locate them while answering some of the questions-

   We should never confuse education with training or the ‘tools’ that educators use. Education is no more a computer or an online class than it is a chalkboard-those are simply tools. Additionally, precious few new and relevant findings have been added to our insights into the learning process; much of what many proclaim to be insightful turns out to be faddish and misguided. What we need for learning to occur are well-prepared and motivated teachers, students who are willing and able to learn and a social system that values educational attainment. Many might take exception to these assertions, raising a host of ancillary social, economic, nutritional and sociological issues. However, decades of data from failed public experiments aimed at mitigating these problems argue to the contrary.

What are we to do ? Create a wave of educational entrepreneurship ? Despite all of the hype and media attention, no one currently knows the best way forward. We must incentivize educationalists, technologists and classroom teachers to experiment and innovate. Such partnerships should freely explore alternatives as we seek to define the blended face-to-face and online classrooms of the future. Some approaches will fall, but that is part of the change process. There will be no onesize-fits-all experience, success will vary by discipline and educational objectives. Unless everyone involved in this process is inspired to take risks we will not enjoy the full potential these new approaches represent. Also, we should not forget the extent to which those who most benefit from the current system will attempt to hijack this change process for their own purposes.

With these issues in mind consider the following vision of the change process to the classroom of tomorrow-a vision that leverages technology to create a more personalized learning experience. With the onset of this vision the professor-student relationship will change. What is not likely to survive is the large class in which everyone progresses at the same pace. That paradigm will be replaced by a more customized and collaborative learning process. The reality of a technology-enabled personalized learning environment is still evolving. What seems clear is that at its heart is a more collaborative and student-mapped and paced process. The function of the professor, aided by educationalists, will also change. Student-professor collaboration will then determine both what content can be assigned to practice methods and how the student demonstrates mastery of that content. In such cases, timing issues will most likely be left entirely in the student’s hands, while technology specialists recommend the best hardware and software solutions. Critical thinking or application ideas will involve more specialized face-to-face and interactive online approaches geared to the real-time needs and progress the student is making. This, the professor, with the student’s help, will vigilantly mix and blend the learning ingredients to produce a new learning environment. How this process plays out in reality will be the result of educational entrepreneurship, but it will surely entail both successes and some failures.

The administration and accreditation of education will also have to change. As more education occurs outside the bricks and mortar framework and is more centred on the student-professor interaction, the role of all nonteaching staff will need to be reassessed. As students and faculty increase their use of technology to personalize their formal and informal learning, educational technologists must be on hand to facilitate the effective use of that technology. Also, the current hierarchical and standardized outcomes formulas for administration and accreditation of colleges and universities are no longer tenable. Realistically, they are an impediment to the change needed.

Finally, we must address one of the most serious challenges facing online education – cheating. Simply put cheating is rampant and we are turning a blind eye to the problem. Cheating threatens the integrity of the educational process and the value-added of the degree. Solutions must be found and implemented or all changes will be for naught.

161. Which of the following is most nearly the same in meaning as the word given in bold as used in the passage?

Entail

(A)  Limit

(B)  Occasion

(C)  Involve

(D)  Subject

(E)   End

Answer: (C)

162. Which of the following can be the most suitable title for the passage?

(A)  Educating through Technology – What the Future can be

(B)  Teachers – An Obsolete Future

(C)  The Hype Around Technology aided Education

(D)  Limitations of Technology

(E)   Evolution of Technology – The Way Forward for Tech Giants

Answer: (A)

163. Which of the following is true according to the author?

(a)   The way the colleges are granted certification will have to change to accommodate the changes in imparting education through technology.

(b)   With the onset of technology enabled education, the school/college premises or building will lose its importance slightly.

(c)   With the onset of the desired level of technology enabled education, learning will become a joint effort of students as well as teachers.

(A)  Only (a)

(B)  All the three (a), (b) and (c)

(C)  Only (b) and (c)

(D)  Only (c)

(E)   Only (a) and (b)

Answer: (B)

164. Which of the following is most opposite in meaning as the word given in bold as used in the passage?

Mitigating

(A)  Aggravating

(B)  Irritating

(C)  Annoying

(D)  Frustrating

(E)   Infuriating

Answer: (A)

165. Which of the following is most nearly the same in meaning as the word given in bold as used in the passage?

Personalize

(A)  Customize

(B)  Cause

(C)  Sentence

(D)  Own

(E)   Preside

Answer: (D)

166. According to the author, which of the following changes word occur with the onset of the desired level of technology aided education?

(a)   The pace of learning for each student will be different.

(b)   The decision of how much time a student needs to learn will be left to the student’s disposal.

(c)   The roe of the teacher will change and become one of being the provider of tailored learning content to the student.

(A)  Only (c)

(B)  Only (b) and (c)

(C)  Only (a)

(D)  All the three (a), (b) and (c)

(E)   Only (a) and (c)

Answer: (A)

167. Which of the following is most opposite in meaning as the word given in bold as sued in the passage?

Impediment

(A)  Freedom

(B)  Advantage

(C)  Extravagance

(D)  Luxury

(E)   Autonomy

Answer: (A)

168. What does the author mean when he uses the words ‘will be for naught’?

(A)  Will be in trouble

(B)  Will face complications

(C)  Will not be allowed

(D)  Will come together

(E)   Will amount to nothing

Answer: (E)

169. According to the author , which of the following is not True about cheating?

(A)  It is one of the gravest problems faced in education

(B)  It undermines the value of the credential obtained through education

(C)  Not much is being done to curtail it at present

(D)  It is quite prevalent at present

(E)   All the given options are true

Answer: (E)

170. According to the author, which of the following should be done to effectively integrate technology into education?

(A)  Risk taking should be kept to a bare mitigate adverse effects.

(B)  Innovation amongst stake-holders of education and technology should be encouraged.

(C)  We should be alert about people who may not allow certain changes for their own benefits.

(A)  Only (a) and (c)

(B)  Only (c)

(C)  Only (b) and (c)

(D)  Only (a)

(E)   All the three (a), (b) and (c)

Answer: (C)

Directions-(Q. 171-175) In this questions four words are given of which two are most nearly same or opposite in meaning. Find the two words which are most nearly the same or opposite in meaning and indicate your answer by marking the option which represents the correct letter combination.

171. (a) Tormented (b) Agonised

(c) Aroused        (d) Removed

(A)  a – c

(B)  a – b

(C)  a – d

(D)  c – d

(E)   b – c

Answer: (B)

172. (a) Striking (b) Unimpressive

(c) Striving         (d) Unwilling

(A)  a – c

(B)  b – d

(C)  a – b

(D)  c – d

(E)   b – c

Answer: (C)

173. (a) Patent (b) Stylish

(c) Wasted          (d) Fashionable

(A)  a – c

(B)  c – d

(C)  a – d

(D)  b – d

(E)   b – c

Answer: (D)

174. (a) Disclose (b) Withstand

(c) Withhold       (d) Decide

(A)  a – c

(B)  b – c

(C)  a – d

(D)  c – d

(E)   b – d

Answer: (A)

175. (a) Fathomed (b) Devastated

(c) Ruined          (d) Abrupt

(A)  b – d

(B)  b – c

(C)  c – d

(D)  a – c

(E)   a – d

Answer: (B)

Directions-(Q. 176-180) The sentence has two blanks. Each blank indicates that something has been omitted. Choose the words that bet fit the meaning of the sentence as a whole.

176. As per the present definition, the factory inspector ….. all factories located in an area to …. a day other than Sunday as the weekly holiday.

(A)  permits, attend

(B)  allows, observe

(C)  assures, believe

(D)  admits, accept

(E)   grants, adhere

Answer: (B)

177. The family members of the victim said that the victim was being … into…. her complaint.

(A)  protected, accepting

(B)  disturbed, locating

(C)  allowed, maintaining

(D)  forced, preparing

(E)   threatened, withdrawing

Answer: (E)

178. The new management is pro-cleanliness and will ….. all the employees who spit or smoke in the office …….

(A)  penalize, hours

(B)  punish, environment

(C)  warn, surrounding

(D)  fine, premises

(E)   pardon, area

Answer: (D)

179. Sixty-six years after India attained independence, the government has finally ….. to change the measure to estimate the output of a steam engine – horsepower that is being ……. since the British Raj.

(A)  thought, operated

(B)  decided, used

(C)  planned, provided

(D)  caused, handled

(E)   reasoned, postponed

Answer: (B)

180. With women participation ……. sharply in the labour force over the past decade, there is a need to improve ………. for them.

(A)  dipping, opportunities

(B)  reducing, ways

(C)  withdrawing, convenience

(D)  stressing, circumstances

(E)   steeping, excuses

Answer: (A)

Directions-(Q. 181-185) Rearrange the given six sentences (1), (2), (3), (4) (5) and (6) in a proper sequence so as to form a meaningful paragraph and then answer the given question.

(1) But it is normal for rates to vary somewhat.

(2) The rate at which these actions emerge is sometimes a worry for parents.

(3) As a child grows, his or her nervous system becomes more mature.

(4) Having said that variation is normal, nearly all children begin to exhibit certain motor skills at a fairly consistent rate unless some type of disability is present.

(5) As this happens, the child becomes more and more capable of performing increasingly complex actions.

(6) Hence, the frequently fret about whether or not their children are developing these skills at a normal rate

181. Which of the following should be the THIRD sentence after the rearrangement?

(A)  1

(B)  2

(C)  3

(D)  5

(E)   6

Answer: (B)

182. Which of the following should be the SIXTH sentence after the rearrangement?

(A)  1

(B)  2

(C)  3

(D)  4

(E)   5

Answer: (D)

183. Which of the following should be the FIRST sentence after the rearrangement?

(A)  1

(B)  2

(C)  3

(D)  4

(E)   5

Answer: (C)

184. Which of the following should be the SECOND sentence after the rearrangement?

(A)  1

(B)  2

(C)  6

(D)  4

(E)   5

Answer: (E)

185. Which of the following should be the FOURTH sentence after the rearrangement?

(A)  1

(B)  4

(C)  2

(D)  6

(E)   5

Answer: (D)

Directions-(Q. 186-190) Read this sentence to find out whether there is any grammatical mistake/error in it. The error, if any, will be in one part of the sentence. Mark the part with the error as your answer. If there is no error, mark ‘No error’ as your answer. (Ignore the errors of punctuation if any).

186. Attributing rise in inflation partly for withholding of food stocks by traders. / the minister said that / he was committed / to easing this supply side bottle-neck.

(A)  Attributing rise in inflation partly for withholding of food stocks by traders

(B)  the minister said that

(C)  he was committed

(D)  to easing this supply side bottleneck

(E)   No error

Answer: (D)

187. India’s largest utility vehicle and tractor maker / is again in the race to acquire / for stake in Swedish company / which is a premium car maker.

(A)  India’s largest utility vehicle and tractor maker

(B)  is again in the rate to acquire

(C)  for stake in Swedish company

(D)  which is a premium car maker

(E)   No error

Answer: (C)

188. With sale of branded or premium petrol becoming almost nil / due to high duties, / a government appointed panel has recommended / slashing excise duty to make them at par with regular fuel.

(A)  With sale of branded or premium petrol becoming almost nil

(B)  due to high duties

(C)  a government appointed panel has recommended

(D)  slashing excise duty to make them at par with regular fuel

(E)   No error

Answer: (B)

189. Keeping in mind / that power cuts are on different days in different areas / the change in the factory law would enable individual factories within an area / to determining their own weekly holidays.

(A)  Keeping in mind

(B)  that power cuts are on different days in different areas

(C)  the change in the factory law would enable individual factories within an area

(D)  to determining their own weekly holidays

(E)   No error

Answer: (D)

190. Police officers have refused on identify the bystander,/ who is the only eyewitness to the crime, / but have said that the investigating team would explore / if he could be a witness in the case.

(A)  Police officers have refused on identify the bystander

(B)  who is the only eyewitness to the crime

(C)  but have said that the investigating team would explore

(D)  if he could be a witness in the case

(E)   No error

Answer: (E)

Directions-(Q. 191-200) In the given passage there are blanks, each of which has been numbered. Against each, five words are suggested, one of which fits the blank appropriately. Find out the appropriate word in each case.

   Generally a disaster …(191)… in significant loss in social, psychological and economic aspects. It not only …(192)… to structural damages, but also leaves families torn part, children orphaned, livelihoods destroyed and communities traumatized.

Non-structural factors such as lack of responsiveness of government officials and ineffective leadership are mainly …(193)… for any disaster mismanagement. Indi is …(194)… to a variety of natural and man-made disasters. Strong and effective emergency management has been a felt …(195)… in all corners of the word. Effective policies play a …(196)… role in mitigating the impact of disasters and reducing likely losses of life and property.

Economic resources are important for any disaster management. Yet, it has been …(197)… that economic resources did not necessarily translate into greater investment in this domain, as there is no …(198)… of issues that demand governments attention and resources. Disaster management has …(199)… acquired importance in the agenda of governance, unless there is a major natural or manmade disaster. The major shortcomings observed in Indian disaster management along with their probable solutions need to be discussed on an …(200)… basis.

191. 

(A)  prepares

(B)  results

(C)  affects

(D)  entails

(E)   promotes

Answer: (B)

192.

(A)  causes

(B)  creates

(C)  results

(D)  imparts

(E)   leads

Answer: (A)

193.

(A)  reasons

(B)  responsible

(C)  accounts

(D)  amounts

(E)   factors

Answer: (A)

194.

(A)  subjected

(B)  susceptibility

(C)  available

(D)  vulnerable

(E)   centre

Answer: (D)

195.

(A)  relation

(B)  abstract

(C)  evolution

(D)  creation

(E)   need

Answer: (E)

196.

(A)  enormous

(B)  tough

(C)  vital

(D)  single

(E)   important

Answer: (C)

197.

(A)  recognized

(B)  routed

(C)  placed

(D)  collected

(E)   seeing

Answer: (A)

198.

(A)  much

(B)  amount

(C)  collection

(D)  dearth

(E)   failure

Answer: (D)

199.

(A)  seldom

(B)  forever

(C)  great

(D)  much

(E)   always

Answer: (E)

200.

(A)  Important

(B)  understanding

(C)  urgent

(D)  priority

(E)   upright

Answer: (C)

© Copyright Entrance India - Engineering and Medical Entrance Exams in India | Website Maintained by Firewall Firm - IT Monteur